Sunteți pe pagina 1din 50

‫גרסה זו ראשונית‪ .‬הגרסה הכי מעודכנת תמיד אפשר לתרגל חינם ב ‪drmada.

com‬‬

‫שחזור בחינת הרישוי ‪02.03.2017‬‬


‫גרסה זו רק למטרת ערעור!‬
‫גרסה סופית ומעודכנת תהיה זמינה ב ‪drmada.com‬‬

‫‪IF you have these questions enter it here http://goo.gl/forms/7fuxBwhJQs‬‬

‫‪or send it to us at https://www.facebook.com/profile.php?id=100010122580724‬‬

‫‪Guide:‬‬
‫‪OQ.03.2017.A.001:‬‬
‫‪OQ.03.2017 = old exam 03.2017‬‬
‫‪A = part A‬‬
‫‪001= Question number 1‬‬

‫תודה לכל הנבחנים שעזרו לצוות שלנו בשחזור הבחינה ומאחלים לכולם בהצלחה‬

‫באהדה רבה‪,‬‬
‫צוות ‪drmada.com‬‬
drmada.com ‫ הגרסה הכי מעודכנת תמיד אפשר לתרגל חינם ב‬.‫גרסה זו ראשונית‬

OQ.03.2017.A.001
Which of the following laboratory results is predictable
predictable of negative prognosis in a 60
60-year-old man with
community-acquired
acquired pneumonia?
A. 13,500 neutrophils/microliter on complete blood count
B. Serum sodium of 121 meq/l
C. 378,000 platelets on complete blood count
D. Serum glucose of 223 mg/dL
-------------------------------------------------------------------------------
OQ.03.2017.A.002
A 52-year-oldold man presents with malaise, shortness of breath, productive cough with foul smelling grey
utum, and a temperature of 38 oC over the last two weeks. Given the X-ray,
sputum, ray, which of the following is
the most likely diagnosis?

A. Lobar pneumonia
B. Atypical pneumonia
C. COPD exacerbation
D. Lung abscess
E. Lung cancer
-------------------------------------------------------------------------------
OQ.03.2017.A.003
A 67-year-oldold woman is admitted for workup of a febrile disease. Staphylococcus MRSA has grown in
three blood cultures. Six days into adequate antibiotic treatment she develops lung edema, and
auscultation reveals a diastolic murmur above the aorta. Which of the following is the next therapeutic
step?
A. Broad spectrum antibiotics in order to cover ggram negative bacteria
B. 5 consecutive days of intravenous Lasix (furosemide)
C. Refer the patient to emergent heart surgery
D. Cease antibiotics and obtain another blood culture
E. Chest CT with IV contrast
-------------------------------------------------------------------------------
OQ.03.2017.A.004
A 27-year-oldold woman presents to the emergency room with a temperature of 40 oC and severe pain over
the right shin. Her systolic blood pressure is 92 mmHg. On examination the shin is very tender on
drmada.com ‫ הגרסה הכי מעודכנת תמיד אפשר לתרגל חינם ב‬.‫גרסה זו ראשונית‬

palpation, but there is no local swelling or erythema of the skin. Complete blood count reveals 42,000
leukocytes/mql. Which of the following
fol is the most likely diagnosis?
A. Necrotizing fasciitis
B. Cellulitis
C. Myositis
D. Erysipelas
E. Stasis dermatitis
-------------------------------------------------------------------------------
OQ.03.2017.A.005
An 82-year-old old woman was referred from the nursing home due to severe diarrhea and high fever. Stool
examination revealed Clostridium difficile (CDT) infection. Her systolic blood pressure is 83 mmHg, and
complete blood count revealed 28,500 leukocytes/mql,
leukocytes/mql, and a creatinine of 2.3 mg/dl. Which of the
following is the best treatment for this patient?
A. Oral metronidazole, 500 mg, 3 times daily
B. Intravenous metronidazole, 500 mg, 3 times daily
C. Oral vancomycin, 125 mg, 4 times daily
D. Intravenous vancomycin, 125 mg, mg 4 times daily
-------------------------------------------------------------------------------
OQ.03.2017.A.006
A usually healthy 20-year-old old female complains of sudden urinary irritation with urgency and increased
frequency. She is not pregnant. What is the best course of management?
A. Urinalysis and cultures and start antibiotic treatment according to results
B. Empiric antibiotic treatment with no further tests
C. Recommend drinking water and rest without antibiotic treatment
D. Refer to a gynecologist before any other testing or treatment
-----------------------------------------------------------------------------------------------
OQ.03.2017.A.007
A 23-year-oldold medical student complains of anxiety before his anatomy exam. Based on the attached
ECG, which of the following is the most likely diagnosis?

A. Sinus arrhythmia, a benign condition in young adults


B. AV block grade 1
C. AV block grade 2, Mobitz 1
D. AV block grade 2, Mobitz 2
E. Premature ventricular beats (VPB’s)
-----------------------------------------------------------------------------------------------
drmada.com ‫ הגרסה הכי מעודכנת תמיד אפשר לתרגל חינם ב‬.‫גרסה זו ראשונית‬

OQ.03.2017.A.008
Which of the following diseases typically causes both stenosis and insufficiency of the mitral valve?
A. Chordae rupture as a complication of acut acute myocardial infarction
B. Acute bacterial endocarditis
C. Dilated cardiomyopathy
D. Rheumatic heart disease
E. Blunt trauma to the chest
-----------------------------------------------------------------------------------------------
OQ.03.2017.A.009
A 62-year-oldold man presents to the emergency room with, malaise and palpitations. His history is positive
for paroxysmal atrial fibrillation. Attached is the ECG script performed immediately.

A. Ventricular tachycardia
B. Ventricular fibrillation
C. Atrial flutter
D. Atrial fibrillation with LBBB
-----------------------------------------------------------------------------------------------
OQ.03.2017.A.010
A patient complains of pain andd erythema of the nose and cheek cheek for several days, and is now complaining
of numbness of part of the face and double vision. Examination reveals reve ls partial paralysis of the eyeball
muscles on the right. What diagnosis should be suspected suspected?
A. Frontal lobe stroke
B. A tumor pressing the optic chiasm
C. Injury to the anterior pituitary gland
D. Injury ry to the posterior pituitary gland
E. Cavernous sinus thrombosis
-----------------------------------------------------------------------------------------------
OQ.03.2017.A.011
A 62-year-oldold woman complains of feeling cold, hair loss, and constipation. Which of the following tests
is appropriate for this patient?
A. ANA antibodies
B. TSH
C. Urine for catecholamines
D. HbA1C
E. Tumor markers CA-125, 125, CEA
drmada.com ‫ הגרסה הכי מעודכנת תמיד אפשר לתרגל חינם ב‬.‫גרסה זו ראשונית‬

-----------------------------------------------------------------------------------------------
OQ.03.2017.A.012
A 52-year-oldold man with advance heart failure (due to several myocardial infarcts)) complains that he is
experiencing enlargement and tenderness in the breast tissue which bothers him over the last month.
Which of the following drugs is responsible for these symptoms?
A. Carvedilol
B. Amiodarone
C. Warfarin
D. Furosemide
E. Spironolactone
-----------------------------------------------------------------------------------------------
OQ.03.2017.A.013
A 44-year-oldold woman recently diagnosed with Burkitt’s Bur lymphoma has begun chemotherapy 3 days ago.
Her laboratory results today reveal increased creatinine, hypophosphatemia, hypocalcaemia hypocalc and
hyperureacemia. Which of the following explains these ph phenomena?
A. Transformation to aggressive leukemia
B. Typical renal injury secondary to chemotherapy
C. Renal injury typical to lymphoma
D. Renal injury due to tumor lysis syndrome
E. Treatment is probably causing rabdomyolisis
-----------------------------------------------------------------------------------------------
OQ.03.2017.A.014
Which of the following is the target hemoglobin value in a patient with chronic renal insufficiency treated
recombinant erythropoietin and iron?
A. 8 gr/dl at the most
B. 10-11,5
11,5 gr/dl at the most
C. Above 12 gr/dl
D. The hemoglobin level vel should be as high as possible
possibl
E. There isn’t a set target; target should be adjusted to each patient separately
-----------------------------------------------------------------------------------------------
OQ.03.2017.A.15
A 28-year-oldold man had a febrile illness with cough for a few days. Two days later he presents with
recurrent hematuria. Which of the following is the most likely diagnosis?
A. IgA nephropathy
B. Post streptococcal glomerulonephritis
C. Lupus nephritis
D. Primary bladder tumor
E. Primary renal malignancy
-------------------------------------------------------------------------------
OQ.03.2017.A.016
A 52-year-oldold woman, with cervical carcinoma presents with fever up to 39 oC, chills, nausea and
vomiting, and systolic blood pressure of 76 mmHg. There is pus in the urine. Blood work reveals renal
insufficiency and an ultrasound scan demonstrates right hydronephrosis. Which of the following is the
most appropriate treatment?
drmada.com ‫ הגרסה הכי מעודכנת תמיד אפשר לתרגל חינם ב‬.‫גרסה זו ראשונית‬

A. Obtain blood and urine cultures and initiate antibiotic treatment


B. Intravenous fluids and nd steroids
C. Urinary catheter and intravenous fluids
D. Drainage via nephrostomy and initiation of empirical antibiotic treatment
E. CT with contrast material
-------------------------------------------------------------------------------
OQ.03.2017.A.017
A 29-year-oldold woman is referred due to bleeding of the oral mucosa and urine. Complete blood count
reveals 32,000 platelets/mql,
/mql, 7900 leukocytes/mql, and a hemoglobin concentration of 12,6 gr%. Blood
smear demonstrated abundant fragmented red blood cells. Which of the following is the most likely
diagnosis?
A. ITP (idiopathic thrombocytopenic purpura)
B. Drug induced thrombocytopenia
C. Congenital thrombopenia
D. Pseudo-thrombocytopenia
thrombocytopenia
E. TTP (thrombotic thrombocytopenic purpura)
-------------------------------------------------------------------------------
OQ.03.2017.A.018
A 27-year-oldold Jewish Ashkenazi male has a tendency for bleeding of oral mucosa. His history is positive
for an unexpected massive bleeding following minor orthopedic surgery. Thos problem exists in other
family members as well. Which of the following clotting factors is responsible for this condition?
A. Factor 7
B. Factor 8
C. Factor 9
D. Factor 10
E. Factor 11
-------------------------------------------------------------
-------------------------------------------------------------------------------
OQ.03.2017.A.019
A 19-year-oldold field soldier complains of high fever, headaches, and a rash on his trunk and limbs. He has
recently been in contact with dogs. Which of the following antibiotics should be initiated even prior to
diagnosis?
A. Amoxycillin
B. Piperacillin
C. Ertapenem
D. Doxycycline
E. Imipenem
-------------------------------------------------------------------------------
OQ.03.2017.A.020
A 62-year-oldold woman, working as a tour guide around the country, presents with bleeding from a mole
located on the right ear lobe. The mole has grown larger recently and has bled after it was scratched.
Which of the following is the next step in the work work-up of this patient?
A. Take a picture of the lesion and follow-up
follow in two weeks
B. Take a picture of the lesion and follow-up
follow in three month
C. Refer to a general/plastic surgeon as soon as possible
drmada.com ‫ הגרסה הכי מעודכנת תמיד אפשר לתרגל חינם ב‬.‫גרסה זו ראשונית‬

D. Recommend applying a steroidal ointment (i.e. dermacombin)


-------------------------------------------------------------------------------
OQ.03.2017.A.021
A 50-year-oldold man is admitted for pyelonephritis. During his hospital stay his pulse goes up to 124
beats/min, ECG shows sinus tachycardia, and rectal temperature is 39.4 oC.
Which of the following describes the next therapeutic step?
A. Treatment with a shortt term beta blocker such as propranolol
B. Treatment with a short term calcium channel blocker such as
C. Treatment with an antipyretic drug such as optalgin
D. Loading does of an antiarrhythmic agent such as amiodarone
E. Intravenous steroids such as hydrocortisone
-------------------------------------------------------------------------------
OQ.03.2017.A.022
A 41-year-oldold woman has been suffering from nausea and severe vomiting for the last two days. She
presents to the emergency room with two episodes of hematemesis. Gastroscopy reveals Mallory-Weiss Mallory
tears. Which of the following is the best treatment for this patient?
A. Conservative treatment, the bleeding is usually self limiting
B. Surgical treatment, suturing of the esophageal tears
C. Endoscopic treatment, ligation
li of the mucosal tears
D. Endoscopic treatment, without adrenaline injection the patient will most definitely continue
bleeding
-------------------------------------------------------------------------------
OQ.03.2017.A.023
A 38-year-oldold heavy smoker, known to have small cell lung cancer, presents with back pain that has
started 3 hours ago, and is aggravated by coughing, sneezing, and lying on his back. Examination reveals
weakness and numbness of the lower extremities and a 350 ml residue in the bladder. Which of the
following is the next therapeutic step?
A. Spine CT prior to any treatment
B. Spine MRT prior to any treatment
C. NSAIDs (such as voltaren) injection prior to imaging
D. Steroid (dexamethasone) injection prior to imaging
ferral to orthopedic surgeon who specializes in spine prior to treatment or imaging
E. Referral
-------------------------------------------------------------------------------
OQ.03.2017.A.024
Which of the following describes the chief complain of patients with interstitial lung disease?
A. Unproductive cough and shortness of breath on exertion
B. Hemoptysis and chest pain
C. Productive cough and febrile illness
D. Weight loss and chest pain
E. Wheezing and respiratory rales
-------------------------------------------------------------------------------
OQ.03.2017.A.025
A 35-year-oldold native Israeli woman, with normal medical history presents with malaise and shortness of
breath. CT demonstrates bilateral perihilar lymphadenopathy,
lymphadenopa and physical examination reveals an
drmada.com ‫ הגרסה הכי מעודכנת תמיד אפשר לתרגל חינם ב‬.‫גרסה זו ראשונית‬

erythematic, nodular, non itching rash over the legs (photograph attached). Which of the following is the
most likely diagnosis?

A. Asthma
sthma with atopic background
B. TBC
C. ANCA associated vasculitis
D. Sarcoidosis
E. SLE
-------------------------------------------------------------------------------
OQ.03.2017.A.026
Which of the following
lowing doesn’t cause clubbing ?
A. lung cancer
B. COPD
C. Cirrhosis
D. IBD
E. lung interstitial disease
-------------------------------------------------------------------------------
OQ.03.2017.A.027
Which
hich of the following change course of COPD patient with serious hypoxia ??
A. short acting B-agonist
agonist inhalation
B. LABA inhalation
C. permanent prophylactic antibiotics such as doxycilline
D. Oxygen
E. regular home physiotherapy
-------------------------------------------------------------------------------
OQ.03.2017.A.028
Which of the following COPD patient fits tthe criteria of stage gold three?
A. FEV1/FVC and FEV1>=30% but <50%
B. FEV1/FVC>0.8 and FEV1>=50% predicted
C. FEV1/FVC<0.5 and FEV1<50 FEV1 but >=10% predicted
D. FEV1/FVC<0.6 and FEV1>=50% but <70% predicted
E. FEV1/FVC >=0.8 and FEV1>=30
-------------------------------------------------------------------------------
drmada.com ‫ הגרסה הכי מעודכנת תמיד אפשר לתרגל חינם ב‬.‫גרסה זו ראשונית‬

OQ.03.2017.A.029
Which of the following treatment is recommended for ascites in cirrhotic patients?
A. Potassium restriction to 8 gr/d
B. Sodium restriction to 2 gr/d
C. Protein restriction to 30 gr/d
D. Calcium restriction to 12 gr/d
E. Chloride restriction to 6 gr/d
-------------------------------------------------------------------------------
OQ.03.2017.A.030
A 40-year-old man has a paracentesis of peritoneal fluid with a serum ascites albumin gradient (SAAG)
score of <1.1 mg%. Which of the following diagnosis is possible in this scenario?
A. Tuberculosis
B. Peritoneal carcinoma
C. Rheumatic arthritis
D. Heart failure
-------------------------------------------------------------------------------
OQ.03.2017.A.031
A 65-year-oldold man, known to have renal failure with creatinine of 2 gr/dl, diabetes, and a gastric ulcer, is
admitted with an acute episode of gout disease. Which of the following treatments is the most appropriate
in this patient?
A. NSAIDs
B. Colchicine
C. Allopurinol
D. Prednisolone
E. AntiTNF
-------------------------------------------------------------------------------
OQ.03.2017.A.032
Which of the following provides the best clinical description of the first acute episodes of gout disease?
A. Appears in the wrist
B. Begins at noon after a heavy meal
C. The episode evolves gradually during the course of two days
D. Untreated, the episode will resolveresolv spontaneously in 3 weeks
E. The clinical presentation may resemble cellulitis
-------------------------------------------------------------------------------
OQ.03.2017.A.033
A 35-year-oldold woman complains of general aches in the neck, shoulder. Flank, and shin areas over the
last 4 month. Additionally, she complains of sleep disturbances, decreased concentration and worsening
fatigue. She denies a rash. CPK, TSH, CRP, ESR and complete blood count are within the normal range.
Which of the following is the most mo likely diagnosis?
A. SLE
B. Sjogren’s syndrome
C. Inflammatory bowel disease
D. Fibromyalgia
E. Hypothyroidism
drmada.com ‫ הגרסה הכי מעודכנת תמיד אפשר לתרגל חינם ב‬.‫גרסה זו ראשונית‬

-------------------------------------------------------------------------------
OQ.03.2017.A.034
An 18-year-old old presents with fever and an inflammatory lesion of the…
You diagnose him with erysipelas. Which of the following bacteria is the most common causative agent?
A. Staphylococcus aureus
B. Staphylococcus epidermitis
C. Streptococcus group A
D. Streptococcus pneumonia
E. Escherichia coli
-------------------------------------------------------------------------------
OQ.03.2017.A.035
A 40-year-old healthy woman diagnosed with a urinary tract infection with Enterococcus faecalis. No
history of previous antibiotic treatment. Which of the following is the most appropriate antibiotic
treatment?
A. Ceftriaxone
B. Gentamycin
C. Amoxicillin
D. Aztreonam
E. Oflaxacin
-------------------------------------------------------------------------------
---------------------------------------------------------------------------
OQ.03.2017.A.036
What is the purpose of head CT in patients in the emergency emergency room with suspected stroke?
A. Diagnose of the extent and location if ischemic cerebrovascular events
B. Ruling out cerebral hemorrhagic event
C. Ruling out brain tumor or metastasis
D. Differentiating a local ischemic event from cardiac emboli
E. Ruling out a frontal stroke
-------------------------------------------------------------------------------
OQ.03.2017.A.037
Which of the following patients would be diagnosed as diabetic?
A. 30-year-old old man, fasting glucose, in two tests on different days 116mg/dl.
B. 41-year-old old asymptomatic man, glucose 190mg/dl on two random tests
C. 37-year-old old man, feeling well,well HbA1C 6.7% in two tests on different days during hospitalization
D. 40- year-old woman,, 2 hours after a glucose tolerance test, blood glucose is 175 mg/dl
E. 36-year-old old man, blood glucose 46mg/dl on a random test
--------------------------------------------
-------------------------------------------------------------------------------
OQ.03.2017.A.038
A 17-year-oldold man, with diabetes type 1 has been complaining of stomach ache, and nausea for last two
days. Blood glucose is 217 mg/dl dl,, bicarbonate is 12mg/, and serum potassium level are 3.3mg/l.
Which of the following must be a part of the treatment in the emergency room?
A. Subcutaneous insulin
B. Intravenous insulin
C. Intravenous empirical antibiotics
D. Intravenous bicarbonate
drmada.com ‫ הגרסה הכי מעודכנת תמיד אפשר לתרגל חינם ב‬.‫גרסה זו ראשונית‬

-------------------------------------------------------------------------------
OQ.03.2017.A.039
Which of the following is considered a risk factor for osteoporosis by the FRAY questionnaire?
A. Male gender
B. Non smoker
C. Rheumatoid arthritis
D. Psoriasis
E. Consuming wine on Fridays
-------------------------------------------------------------------------------
OQ.03.2017.A.040
Which of the following drugs warrants complete blood count follow up?
A. Aspirin
B. Plavix
C. Optalgin
D. Methimazole
E. Penicillin G
-------------------------------------------------------------------------------
OQ.03.2017.A.041
Which of the following drugs may cause red man syndrome?
A. Chloramphenicole
B. Doxycycline
C. Vancomycin
D. Metronidazole
E. TMP-MTX
-------------------------------------------------------------------------------
OQ.03.2017.A.042
A 70 year-oldold man complains of malaise, which gets worse during the evening and night. He has
difficulty in opening his eyes and shaking hands during the night. night. He has recently begun a work-up
work of
anterior mediastenal mass.
Which of the following diagnosis fists this association?
A. Brain metastasis of lung cancer
B. signs of lymphproliferative disease
C. Advanced tuberculosis
D. Myasthenia gravis due to thymoma
E. Malignant thyroid tumor
-------------------------------------------------------------------------------
OQ.03.2017.A.043
A 50-year old man with a mid-systolicsystolic crescendo-de-crescendo murmur.
Which of the following heart defects cause this kind of murmur?
A. Mitral insufficiency
B. Mitral Stenosis
C. Aortic insufficiency
D. Aortic Stenosis
E. VSD
drmada.com ‫ הגרסה הכי מעודכנת תמיד אפשר לתרגל חינם ב‬.‫גרסה זו ראשונית‬

-------------------------------------------------------------------------------
OQ.03.2017.A.044
Which of the following findings on neurologic examination points to a problem originating in the
Substania nigra?
A. Cogwheel rigidity
B. Dysmetria
C. Intubation
D. Myoclonus
-----------------------------------------------------------------------------------------------
OQ.03.2017.A.045
Which of the following is correlated with the disease severity iin n inflammatory bowel diseases?
A. Pyoderma gangrenosum
B. Erythema nodosum
C. Posterior Uveitis
D. Alopecia
E. Sacroileitis
-------------------------------------------------------------------------------
OQ.03.2017.A.046
A 72-year old man with long lasting diabetes. He is feeling well and is afebrile.
Attached is a photograph of his right foot. Which of the following is the most likely diagnosis of patient?

A. Septic arthritis
B. Cellulitis
C. Charcot arthropathy
D. Club foot
-----------------------------------------------------------------------------------------------
OQ.03.2017.A.047
Which of the following mutations is common in myeloproliferative diseases such as polycythemia vera?
A. EGRF
B. HER 2
C. VEGF
D. JAK2
E. P53
-----------------------------------------------------------------------------------------------
drmada.com ‫ הגרסה הכי מעודכנת תמיד אפשר לתרגל חינם ב‬.‫גרסה זו ראשונית‬

OQ.03.2017.A.048
Which of the following is an absolute indication for thrombolysis in patients with pulmonary emboli?
A. Room air oxygenxygen saturation of 78%
B. Heart rate of 122 bmp
C. Systolic blood pressure of 74 mmHg
D. Signs of right ventricular
entricular strain on echocardiography
E. Patient complains of compressing chest pain by the patient
-----------------------------------------------------------------------------------------------
OQ.03.2017.A.049
Which of the following IS NOT considered a risk factor for obstruction sleep apnea?
A. Obesity
B. Female gender
C. Micrognathia
D. Acromegaly
E. Family history of sleep apnea
-----------------------------------------------------------------------------------------------
OQ.03.2017.A.050
Which of the following is the treatment of choice for skin candidiasis?
A. Fluconazole tablets
B. Topical azole
C. Caspofungin
D. Amphotericin B
----------------------------------------------------------------------------
OQ.03.2017.A.051
A woman on her 39th week of gestation presents to the emergency room with prolapse of cord. Which of
the following statements the most accurate regarding the common situation in which this would occur, occur
and the
he most appropriate management?
A. Cord prolapse is more common in term vs vs. preterm babies, and it requires an emergency cesarean
section
B. Cord prolapse pse is more common in preterm babies, and the most appropriate management is
returning the cord to the uterine cavity
C. Cord collapse is more common in vertex presentation vs. vs transverse or footling breech, and it
requires and emergency cesarean section
D. Cord presentation and it delivering the baby as soon as
rd collapse is more common in vertex presentations,
possible
E. Cord collapse is more common in term babies babies, and requires artificial rupture of membranes and
instrumental delivery
-------------------------------------------------------------------------------
OQ.03.2017.A.052
A 38 week pregnant woman presents present with headaches, BP of 155/115, and +3 protein on urinary exam.
Which
hich of the following is most important next step in managing this patient?
A. Magnesium sulfate treatment while inducing labor
B. Medical
edical management of the hypertension, awaiting spontaneous delivery
C. Emergency cesarean section
drmada.com ‫ הגרסה הכי מעודכנת תמיד אפשר לתרגל חינם ב‬.‫גרסה זו ראשונית‬

D. Immediate
mmediate treatment with magnesium, corticosteroids and nitropruside,, and induction of labor
E. Brain imaging as soon as possible
-------------------------------------------------------------------------------
OQ.03.2017.A.053
A woman on her 28th week of gestation is treated with magnesium sulfate in the high high-risk pregnancy unit
due to premature contractions and cervical effacement to 4mm. The womann complain complains of extreme fatigue
and shortness of breath, and on physical examination you find complete loss of patellar reflexes. Which of
the following is the immediate therapeutic
herapeutic next step?
A. Emergency cesarean section
B. Switching from magnesiumagnesium treatment to indomed
C. IV calcium gluconate
D. Prepare for intubation and transfer to ICU
E. IV hypertonic saline
-------------------------------------------------------------------------------
OQ.03.2017.A.054 (OQ.09.2015.A.065)
A 39-year-old woman on her 24th th week of her 3rd pregnancy. Her blood type is B B- and she had a baby
with hydrops fetalis on her second pregnancy. How ow should her current pregnancy be managed?
A. Deliver
eliver the baby at 34 week of gestation to avoid similar situation
B. Follow middle cerebral artery flow from the 18th week of gestation estation onward
C. Biophysical
iophysical profile from the 23th week of gestation onward
D. Uterine
terine artery flow follow up from the 16th week of gestation onward
E. follow up maternal antibody titer and administer 300 mcg of anti-D anti D if necessary
-------------------------------------------------------------------------------
OQ.03.2017.A.055 (OQ.09.2015.A.062)
Which of the following is the definition of a normal non stress test?
A. Three accelerations of at least 10 seconds of 15 bpm above baseline du ring a time period of 20
during
minutes
B. One acceleration of at least 20 seconds of 20 bpm above baseline during a time period of 20
minutes
C. Fetal monitoring with no decelerations over at least 25 minutes
D. Two accelerations of at least 10 seconds of 20 bpm above baseline during a time period of 30
minutes
E. Two accelerations of at least 15 seconds of 15 bpm above baseline during a time period of 20
minutes
-------------------------------------------------------------------------------
OQ.03.2017.A.056
A 21-year-oldold woman, 26 weeks pregnant describes a round, erythematosus,, elevated, and non-painful non
ulcer that has appeared six weeks earlier in the proximity of the vaginal opening. She notes having
unprotected sex with several partners over the last year. Which of the following would you expect on a
fetal sonogram?
A. Hydronephrosis and hyper-echogenic
hyper bowel
B. Severe oligohydramnion
C. Hepatosplenomegaly, polyhydramnion, and an enlarged placenta
D. Chorioretinitis and brain calcifications
drmada.com ‫ הגרסה הכי מעודכנת תמיד אפשר לתרגל חינם ב‬.‫גרסה זו ראשונית‬

E. An atrophic, thin placenta and a fetal microcephaly


-------------------------------------------------------------------------------
OQ.03.2017.A.57
Which of the following DOES NOT increase the risk for post partum hemorrhage?
A. Brief labor
B. Prolonged labor
C. Grand multipara
D. Chorioamnionitis
E. Oligohydramnion
-------------------------------------------------------------------------------
OQ.03.2017.A.058
In all the following conditions a pregnant woman w will get vaccinated, except:
A. Tetanus vaccine to a 14 weeks pregnant woman that cut herself from a rusty sharp iron piece in
the street
B. A flu vaccine for a 27 weeks pregnant woman with chronic asthma
C. A VZV vaccine for a 13 weeks pregnant woman exposed to her sick son
D. A pertussis vaccine for a 32 weeks pregnant woman working as a nurse in a clinic
E. An inactivated polio vaccine for a 22 weeks pregnant woman during a polio epidemic
-------------------------------------------------------------------------------
OQ.03.2017.A.059
A woman with SLE, which had intra uterine fetal death at 27 weeks on her previous pregnancy, comes for
follow up on her 8th week of gestation. Two pre-gestational
pre gestational laboratory tests were positive for lupus
anticoagulant antibodies, 3 month apar apart.t. Which of the following describes the best management for this
woman?
A. Aspirin and Coumadin treatment through the pregnancy
B. Plasmapheresis every 2 weeks
C. Prednisolone treatment through the pregnancy
D. Aspirin and heparin treatment through the pregnancy
E. Colchicine and plaquanil treatment through the pregnancy
-------------------------------------------------------------------------------
OQ.03.2017.A.060
The woman described in the previous question is at risk for all of the following complications
co EXCEPT:
A. Pre-eclampsia
B. Intrauterine growth retardation
C. Post-term labor
D. Early abortion
E. Pulmonary embolism
-------------------------------------------------------------------------------
OQ.03.2017.A.061
Which of the following, regarding the course and length of labor, may follow epidural anesthesia?
A. Prolongation of the 2nd stage of labor
B. Prolongation of the 1st stage of labor
C. Decreased risk of instrumental delivery
D. Shortening of the 1st stage of labor
drmada.com ‫ הגרסה הכי מעודכנת תמיד אפשר לתרגל חינם ב‬.‫גרסה זו ראשונית‬

E. Increased risk of fetal asphyxia


-------------------------------------------------------------------------------
OQ.03.2017.A.62
Which of the following is the most suitable candidate for trail of vaginal labor after cesarean (TOLAC)?
A. A woman post CS with a longitudal uterine incision at 28 28 weeks of gestation
B. A woman post CS due to breech presentation, with a low horizontal uterine incision
C. A woman post two elective CSs, with an unremarkable post operative history
D. A woman post a CS when fully dilated due to a diagnosis of cephalo pelvic disproportion
E. A woman post elective CS due to breech presentation, who requires induction for post term
pregnancy
-------------------------------------------------------------------------------
OQ.03.2017.A.63
year old couple is married for a year and having unprotected sex. They have come for infertility
A 22-year
work up, and have been sent for a postcoital test. Which of the following describes the post coital test’s
contribution to infertility work up?
A. it may be used as an alternative for spermogram and is an essential part of the work up
B. need to be performed only if mechanical failure is suspected
C. its value in this couples infertility evaluation is questionable
D. contributes to the definition of the severity of the spermatic defect
E. essential for evolution normal
nor peripheral estradiol function
-------------------------------------------------------------------------------
OQ.03.2017.A.64
A 35-year-oldold couple wishes to conceive, the man does not come home every night, they wish to follow
the woman ovulation cycle, so they can time their itercourse to point where chances of conceiving are
maximal. Which hich of the following test is the most accurate in their timing their intercourse
inte in order to
conceive?
A. serum prolactin >5 ng/ml
B. LH increase in serum or urine
C. pelvic ic fluid on uterine sonogram
D. serum estradiol >120 g/ml
E. sonogram demonesterid and leading follicle with a 14 mm diameter
-------------------------------------------------------------------------------
OQ.03.2017.A.065
An infertility work up for a 32-year year-old couple revealed: the woman has a 40-day day- menstural cycle, serum
progesterone was 1ng/ml on day 23, 23 hysterogram showed a normal uterine cavity with contrast material
passing bilaterally, and pelvic diffusion sperm test revealed a concentration of 15 million mil cells/ml, 40%
morphologhy Which of the followingg is the most likely diagnosis?
motility, and 4 % normal Kruger morphologhy.
A. infertility due to corpus luteum insufficiency
B. infertility due to male factor
C. infertility
tility due to mechanical factor
D. idiopathic infertility
E. infertility due to a menstrual cycle abnormality
-------------------------------------------------------------------------------
OQ.03.2017.A.66
drmada.com ‫ הגרסה הכי מעודכנת תמיד אפשר לתרגל חינם ב‬.‫גרסה זו ראשונית‬

A 35-year-oldold single woman wishes to evaluate her ovarian reserve, so she can decide at what age to
conceive. Which of the following predicts a low ovarian reserve?
A. Corpus luteum on the 22nd day of the menstrual cycle
B. A 27-day day long menstrual cycle
C. Serum progesterone of 3-6 3 ng/ml on day 22 of the menstrual cycle
D. LH level higher than FSH level on the 14th day of the menstrual cycle
E. Increased serum estradiol on day 3 of the menstrual cycle
---------------------------------------------------------------------
-------------------------------------------------------------------------------
OQ.03.2017.A.67
A 28-year-oldold woman wishes to conceive. Work up of the couple reveals: normal sperm test, and a
menstrual cycle of 45-60 60 days. She was offered treatment with clomiphene citrate, for ovulation
induction. Which of the following statements is true regarding the chances and risks of this treatment?
A. There is a 25% risk of a multiple embryo pregnancy
B. There is a 75% chance of conceiving every treatment cycle
C. There is a 95% chance of achieving ovulation in every treatment cycle
D. There iss an increased risk of blurred vision and emotional lability
E. The risk for a multiple embryo pregnancy exists only if the patient will also receive HCG
-------------------------------------------------------------------------------
OQ.03.2017.A.68
A 35-year-old woman was referred for hysterosalpinography
hystero as a part of a check
k up of suspected
mechanical infertility; she was offered to perform the test with aqueous or oily contrast material material. Which
of the following is true regarding the comparison of these low contrast materials?
A. aqueous contrast provide better imaging of the salpinx vs. oily contrast material
B. there is greater chance of conceiving after imaging with oily contrast material vs. aqueous material
C. aqueous contrast provide better imaging of the the uterine cavity vs. oily aqueous contrast material
D. there is a decreased risk of granuloma following imaging with oily contrast material vs. aqueous
contrast material
E. there is a slightly greater risk of abdominal pain after imaging with aqueous contrast contr vs. oily contrast
material
-------------------------------------------------------------------------------
OQ.03.2017.A.69
Which
hich of the following treatment is the most appropriate for a 40-year-old old woman with severe
endometriosis who wishes to conceive?
A. GNRH agonist for a year
B. combined oral contraceptive for a year
C. aromatase inhibitors for a year
D. adhesiolysis and lesion ablation
E. IVF
-------------------------------------------------------------------------------
OQ.03.2017.A.70
Which of the following is a characteristic of the dermatologic changes during menopause?
A. subcutaneous edema
B. decreased in elastin and increase in collagen
C. increased blood vessels
drmada.com ‫ הגרסה הכי מעודכנת תמיד אפשר לתרגל חינם ב‬.‫גרסה זו ראשונית‬

D. epidermal flattening
E. skin erythema
--------------------------------------------
-------------------------------------------------------------------------------
OQ.03.2017.A.71
Which of the following is true regarding intra cytoplasmic sperm injection (ICSI)?
A. Is necessary when a defrosted sperm is used to fertilize the ova
B. Is necessary when pre implantation molecular diagnosis is desired
C. There is no need for it if he sperm is motile
D. Performed only when the sperm used was produced from testes
E. Is routinely performed in patients over 40
-------------------------------------------------------------------------------
OQ.03.2017.A.72
A 26-year-oldold woman who has had unprotected sex 48 hours previously is interested in contraception.
She has an unremarkable history, a 28-day 28 day menstrual cycle, and her last menstruation was 21 days ago.
Which of the following is the best way to avoid pregnancy in this woman?
A. The timing of the intercourse does not necessitate intervention
B. Recommend regular use of combined oral contraceptives
C. Recommend regular use of progesterone only oral contraceptives
D. Recommend high dose levonogestrel
-------------------------------------------------------------------------------
---------------------------------------------------------------
OQ.03.2017.A.73
A 35-year-oldold married woman, mother of three, with an unremarkable medical, surgical, obstetrical and
gynecological history wishes to place an intrauterine device (IUD) for contraception. She is debating
whether to use a copper or a progesterone secreting device. Which of the following describes the
difference between these two devices?
A. There is a higher risk of ectopic pregnancies with the progesterone secreting device vs. the copper
device
B. Over time, the progesterone secreting device is associating with ddecreased
ecreased bleeding vs. the cooper
device
C. There is a decreased risk for pelvic inflammatory disease with copper vs. progesterone secreting
device
D. The copper device is a significantly less efficient contraceptive
E. The progesterone secreting device is efficient for a bigger period of time compared to the copper
device.
-------------------------------------------------------------------------------
OQ.03.2017.A.74 (SIMILARSIMILAR to OQ.09.2015.A.084)
An 82-year-old old diabetic woman, with history of an ischemic heart disease and renal failure, presents with
prolapse of the pelvic organs that significantly deteriorates her quality of life. Examination reveals uterine
prolapse grade 3. Which of the following is the most appropriate treatment?
A. An anterior colporraphy operation
B. Transvaginal hysterectomy
C. Antimuscarinic therapy
D. A combined anterior-posterior
posterior operation
E. Local estrogen and use of a passary
drmada.com ‫ הגרסה הכי מעודכנת תמיד אפשר לתרגל חינם ב‬.‫גרסה זו ראשונית‬

-------------------------------------------------------------------------------
OQ.03.2017.A.75 (OQ.02.2016.B.101)
A 38-year-oldold woman, mother of 3, presents with leakage of urine when coughing, laughing, and
sneezing. She denies urgency, frequency, or a burning sensation while urinating. Which of the following
is the recommended initial therapy?
A. A TVT tape
B. Collage injection to the bladder neck
C. Burch colposuspension
D. Anticholinergic treatment
E. Pelvic floor exercises
-------------------------------------------------------------------------------
OQ.03.2017.A.76
A 66-year-old woman oman went through curettage due to postmenopausal bleeding. Pathology revealed
papillary serous adenocarcinoma of the endometrial mucosa. Which of the following is the most
appropriate treatment for this patient?
A. Aromatase inhibitors
B. A radical hysterectomy including salpino-oophrectomy
salpino
C. High dose progesterone for 6 month months
D. Mirena ( intra uterine device secreting levonogestrel )
E. A hysterectomy,
ysterectomy, salpino-oophorectomy,
salpino plus removal of the omentum and lymph nodes
-------------------------------------------------------------------------------
OQ.03.2017.A.77 (OQ.10.2016.B.011)
Which of the following is considered a protective factor from epithelial ovarian cancer?
A. Early menorrhea
B. Late menopause
C. Oral contraceptives
D. Endometriosis
E. Caucasians
------------------------------------------------------------------------
-------------------------------------------------------------------------------
OQ.03.2017.A.78
A 34-year-old woman on the 7th week of her 3rd pregnancy presents with hyperemesis gravidarum. Beta
HCG is 1,000,000. Which of the following is typical of the most likely diagnosis?
A. Maternal bradycardia
B. The uterus is smaller than expected for gestation
C. Spider hemangioma of the skin
D. Hypotension
E. Snow storm image on sonogram
-------------------------------------------------------------------------------
OQ.03.2017.A.79
Which of the following is true regarding the intermediate fluid and the use of energy in hysteroscopy?
A. When using bipolar energy saline should not be used
B. When using bipolar energy glycin may be used as the intermediate fluid
C. Uterine perforation is more common when using bipolar energy
D. Hypokalemia is a common complication of using glycin
drmada.com ‫ הגרסה הכי מעודכנת תמיד אפשר לתרגל חינם ב‬.‫גרסה זו ראשונית‬

E. Manitol is the most common fluid when bipolar energy is used


-------------------------------------------------------------------------------
OQ.03.2017.A.80
Which of the following is a characteristic of diagnosis, implication aand nd treatment of uterine leiomyoma
during pregnancy?
A. It is hard to diagnose by sonogram and requires MRI for diagnosis
B. In cases of torsion or degeneration of the leiomyoma, emergency surgery is required
C. Increases the risk of placenta previa and cesarean section
D. The majority of women with leiomyoma during pregnancy experience pain
E. More common in Caucasian women than in African American women
-------------------------------------------------------------------------------
OQ.03.2017.A.81
A 26-Year-old old woman with a history of pelvic inflammatory disease and chronic pelvic pain undergoes
exploratory laparoscopy reveals violin string adhesions, near the liver. Which of the Following is the most
likely cause of inflammation?
A. Group A Streptococcus
B. Chlamydia Trachomatis
C. Gardenella vaginalis
D. E.Coli
E. Enterococcus Faecalis
-------------------------------------------------------------------------------
OQ.03.2017.A.82 (OQ.02.2016.B.103)
A 19-year-oldold women presents with severe vaginal pain and a burning sensation.
Examination reveals vesicular lesions near the vaginal opening and on the right labia minor, and there is
lymphadenopathy in the groin. Which W ich of the following is the recommended treatment?
A. Acyclovir
B. Metronidazole
C. Cefazolin
D. Fluconazole
E. Betamethasone
-------------------------------------------------------------------------------
OQ.03.2017.A.83 (OQ.02.2016.B.100)
A 32-year-oldold woman presents with a foul-smelling
foul non-itching
itching vaginal secretion. Examination reveals a
grey, homogenous, foul smelling vaginal secretion. The smell intensities after mixing with KOH K solution
and Clue Cells are observed under the microscope. Which of the following is the most appropriate
treatment?
A. First generation Cephalosporin
B. A Single dose of Fluconazole
luconazole 150 mg
C. Vaginal washes with intimate 500P
D. Metronidazole
E. Amoxicillin
-------------------------------------------------------------------------------
OQ.03.2017.A.84
drmada.com ‫ הגרסה הכי מעודכנת תמיד אפשר לתרגל חינם ב‬.‫גרסה זו ראשונית‬

A 22-Year-old old woman presents with an itching vaginal secretion. Examination reveals a white odorless
secretion, which looks like cottage cheese. Which of the following is the most appropriate treatment?
A. Ciprofloxacin for 1 Week
B. A Single dose of Fluconazole
C. Vaginal washes with an intimate soap for 1-week 1
D. A Single dose of metronidazole
E. Penicillin G for 3 days
-------------------------------------------------------------------------------
OQ.03.2017.A.85
A 27-Year-old woman presents with pain and swelling of the vulva. Examination reveals a woman in
distress, fever up to 38.5 C. and you palpable a tender, swollen, fluctuating mass in the lower third of the
right labia majora. Which of the following is the most common cause of this phenomenon?
A. An obstructed glandular duct
B. Infection of a hair follicle
C. Recto-vaginal fistula
D. Urethral infection
E. local trauma
-------------------------------------------------------------------------------
OQ.03.2017.A.86
A 41 year old man reports he tends to look back constantly, worried he might have dropped a coin or
stepped on an insect and killed it. He avoids touching objects, and when he does touch an object, he needs
to convince himself he didn't break it. His sole passion in life is cleaning his house, which he does daily.
Which of the following is the most likely diagnosis?
A. Paranoid personality disorder
B. Acute and transient paranoid psychosis
C. Obsessive compulsive disorder
D. Schizophrenia
E. Hypochondriasis
-------------------------------------------------------------------------------
OQ.03.2017.A.87
A 50 years old was arrested for beating his wife. In his questioning he claimed he is convinced she is
cheating on him with the neighbor, since he found traces of semen in her underwear. He claims he has
following her for a long time, checked her phone and found calls from an unidentified caller, which
inced him beyond doubt that she talks to her love. Which of the following is the most likely
convinced
diagnosis:
A. delusional disorder - erotomanic type
B. delusional disorder - grandiose type
C. delusional disorder - jealous type
D. delusional disorder - persecutory type
E. delusional disorder - addictive type
-------------------------------------------------------------------------------
OQ.03.2017.A.88
Which of the following is considered a type of psychotherapy:
A. ECT
drmada.com ‫ הגרסה הכי מעודכנת תמיד אפשר לתרגל חינם ב‬.‫גרסה זו ראשונית‬

B. cognitive behavioral therapy


C. pharmacotherapy
D. massage
-------------------------------------------------------------------------------
OQ.03.2017.A.89
A 75 years old complains of insomnia, has lost 10 kg over the last couple of month and expresses great
sadness and despair. He has been crying a lot, and has lost interest in friends and social activities. On
examination he is dressed in black and says he doesn't see any point in living. Extensive blood work and
imaging test was normal. Which hich of the following is the most appropriate treatment:
A. haloperidol 10mg x1/d
B. citalopram 20mg x1/d
C. resprim forte 1 tab x1/d
D. methylphenidate 10mg x1/d
E. topiramate 100mg x1/d
-------------------------------------------------------------------------------
OQ.03.2017.A.090
Which of the following
lowing is an indication for ECT?
A. psychotic feature in Alzheimer disease
B. sadistic personality disorder
C. major depression with psychotic feature
D. cannabis intoxication
-------------------------------------------------------------------------------
OQ.03.2017.A.091
A 25 years old man has been brought by an ambulance to the the emergency room for wandering around
the city naked while mumbling biblical passages. On examination in the ER he is found to be dirty and
neglected, staring blankly,, with a flat affect. He claims he has heard the voice of god ordering him to save
the people of Israel of their sins. And since he is so unique there are people who wish to invade his
thought and plant flowers in them. Which hich of the following the most likely diagnosis:
A. OCD
B. major depressive disorder
C. schizophrenia
D. ADHD
E. gender dysphoria
-------------------------------------------------------------------------------
OQ.03.2017.A.092
A 27 years old complains she has tearing off her hair and eyebrows regularly. This behavior has begun
when she was 11 years old and used to tear the hair off her nap. She has been growing out her hair to hide
the bald spot in her occipital bald area. Which of the following is the most probable diagnosis for this
patient:
A. excoriation (skin-picking)
picking) disorder
B. hair pulling disorder (trichotillomania)
C. post traumatic stress disorder
D. depersonalization disorder
E. histrionic personality disorder
drmada.com ‫ הגרסה הכי מעודכנת תמיד אפשר לתרגל חינם ב‬.‫גרסה זו ראשונית‬

-------------------------------------------------------------------------------
OQ.03.2017.A.93
Which
hich of the following describes the difference between dementia and delirium:
A. the thoughts in delirium are slow, vs. racing thought in dementia
B. the thoughts in delirium are disorganized, vs. impoverished thoughts in dementia
C. delirium is characterized by normal awareness vs. reduced awareness in dementia
D. delirium is characterized by significant long term memory impairment vs. preserved memory in
dementia
-------------------------------------------------------------------------------
OQ.03.2017.A.094
A 32-year-oldold lawyer presents to the clinic complaining of distress that began following the relocation of
his law office to a place further in town, which forces him to drive to work. He describes a significant fear
of driving, entering in the car makes him scared he will be killed in a car accident, which causes
tachycardia , shortness of breath and cold sweat. He tried to drive, but was forced to stop for vomiting.
What is the most likelykely diagnosis?
A. Acute psychotic episode
B. Specific phobia
C. Social anxiety disorder
D. General anxiety disorder
E. Substance anxiety disorder
-------------------------------------------------------------------------------
OQ.03.2017.A.095
Which of the following is a symptom of catatonia?
A. Catalepsy
B. Visual hallucination
C. Negative symptoms
D. Sad mood
E. Delusions of grandeus
-------------------------------------------------------------------------------
OQ.03.2017.A.096 (OQ.10.2016.B.031)
Which of the following IS NOT a positive prognostic sign in schizophrenia?
A. Mood disorder
B. Marriage
C. First psychotic break after the age of 40
D. A strong familial support system
E. Negative symptoms
-------------------------------------------------------------------------------
OQ.03.2017.A.097
Brain computerized tomography in schizophrenic patients may reveal:
A. Lateral and third ventricle dilatation
B. Significantly increased grey matter
C. Brain stem asymmetry
D. Significantly increased limbic system structures
-------------------------------------------------------------------------------
drmada.com ‫ הגרסה הכי מעודכנת תמיד אפשר לתרגל חינם ב‬.‫גרסה זו ראשונית‬

OQ.03.2017.A.98
A 58-year-oldold man presents to the emergency room with visual hallucinations of angels hovering aside
him wherever he goes. On examination he reveals he has been suffering from headaches, which awake
him from asleep over the last two month. He is generally healthy, this is the first episode, and he has no
psychiatric history, and takes no regular medications.
First step in the patient management:
A. Start perphenazine
rphenazine 32 mg * 1/d urgently
B. Request compulsory admission from the district psychiatrist
C. Brain imaging
D. Mini mental state examination
----------------------------------------------------------------------------
OQ.03.2017.A.99
Which of the following is true regarding patients diagnosed with PTSD?
A. 30% make full recovery
B. 50% experience severe symptoms throughout their life without remission
C. After 1 year 90% will have mild symptoms
D. After 5 years 10% will have full recovery
-------------------------------------------------------------------------------
OQ.03.2017.A.100
A 26-year-oldold woman is experiencing daily bouts of extreme overeating, followed by guilt , intention
vomiting and a daily run of 5 km. Her BMI is 24.
What is the most likely diagnosis?
A. Anorexia nervosa
B. Bulimia nervosa
C. Body dismorphic disorder
D. Somatic symptoms disorder
-------------------------------------------------------------------------------
OQ.03.2017.A.101
hich of the following drugs has the lowest incidence of extrapyramidal adverse effect ?
Which
A. leponex (clozapine)
B. halidol (haloperidol)
C. respiridol (respiridone)
D. perphanen (Perphenazine)
-------------------------------------------------------------------------------
OQ.03.2017.A.102
A 36-year-oldold man presents to the emergency room with chest pain. He was convinced he was having a
heart attack, complained of feeling light headed and was sweating. An examination by a cardiologist and
an internal medicine doctor was normal, and he was sent sent for a psychiatric evaluation. His history revealed
th
that this was his 6 visit to the ER with similar complains over the last 2 month. Which of the following
is the most likely diagnosis?
A. Somatic symptom disorder
B. Panic disorder
C. Pain disorder
D. Factitious disorder
drmada.com ‫ הגרסה הכי מעודכנת תמיד אפשר לתרגל חינם ב‬.‫גרסה זו ראשונית‬

E. Illness anxiety disorder


-------------------------------------------------------------------------------
OQ.03.2017.A.103
A 62-year-oldold man was admitted for orthopedic surgery. On the 3rd day of his hospital stay, prior to the
surgery, he began experiencing visual hallucination of a little animals and people walking around the
room. These hallucinations appeared at night. Which is the most likely explanation for these symptoms?
A. Cocaine abuse
B. Head trauma
C. Cannabis use
D. Alcoholism
E. Opiate use
-------------------------------------------------------------------------------
OQ.03.2017.A.104
A 34-year-oldold man was brought in for examination. He claimed he is positive he saw through other
people’s eyes the need to be brought to the “light” for cure. In his examination he repeated the same
expressions in a vague and concrete manner, used the word “hasmanus” without explaining what it
means, jumped from subject to subject with no logic connection, and it was very difficult to follow what
he was saying. All of the following exist in his status EXCEPT:
A. Loose associations
B. Neologism
C. Delusions
D. Blocking
E. Stereotyped behavior
-------------------------------------------------------------------------------
OQ.03.2017.A.105
Which of the following is the worst prognostic factor in schizophrenia?
A. Negative symptoms
B. Affective symptoms
C. Sudden onset
D. Late onset
E. Positive symptoms
-------------------------------------------------------------------------------
OQ.03.2017.A.106
In which of the following is the highest rate of metabolic syndrome?
A. Haloperidole
B. Olanzapine
C. Risperidone
D. Perphenazine
D. Ziprazidone
-------------------------------------------------------------------------------
OQ.03.2017.A.107
Which of the following IS NOT a risk factor for suicide?
A. Male gender
B. Low socioeconomic status
drmada.com ‫ הגרסה הכי מעודכנת תמיד אפשר לתרגל חינם ב‬.‫גרסה זו ראשונית‬

C. Immigrants
D. Chronic pain
E. Alcohol abuse
-------------------------------------------------------------------------------
OQ.03.2017.A.108
In patients with mental disorders, which of the following is the best predictor for violent behavior?
A. A history of violence
B. Drug abuse
C. Following therapeutic recommendations
D. Unexpected stressors
E. The type and the course of the mental illness
-------------------------------------------------------------------------------
OQ.03.2017.A.109
Which of the following is the best predictor of suicide in hospitalized psychiatric patients?
A. A history of suicidal attempts
B. The number of past hospitalizations
C. Stress
D. The length of the current hospitalization
E. Compliance to treatment
-------------------------------------------------------------------------------
OQ.03.2017.A.110
A 70-year-oldold man has started antidepressant medication last week. He complains of severe drowsiness.
Which of the following drug is most likely to cause this phenomenon?
A. Duloxetine (cymbalta)
B. Bupropion (wellbutrin XR)
C. Fluoxetine (prozac, prizma, flutine)
D. Escitalopram (cipralex, esto)
E. Mirtazapine (miro, remeron)

Part B
OQ.03.2017.B.001
Prolonged use of Lithium (licarbium) may cause all of the following except
A. Slight hand tremor
B. Weight gain
C. Renal injury
D. Thyroid injury
E. Hepatic injury (cirrhosis)
-------------------------------------------------------------------------------
OQ.03.2017.B.002
Which tests and why is it important to perform monthly blood tests in patient receiving Clozapine
(leponex)?
A. Complete blood count due to risk for thrombocytopenia
drmada.com ‫ הגרסה הכי מעודכנת תמיד אפשר לתרגל חינם ב‬.‫גרסה זו ראשונית‬

B. Leponex levels, for follow up on blood levels


C. Leponex levels, due to a narrow therapeutic index, in order to avoid intoxication
D. Complete blood count, due to the risk for agranulocytosis
E. Blood chemistry, due to the risk for hepatic injury
-------------------------------------------------------------------------------
OQ.03.2017.B.003
Which of the following instructions is important for a patient taking Lamotrigine (lamictal)?
A. Liver function tests every 3 month
B. Immediate referral to a doctor is a rash appears
C. ECG prior to the initiation of therapy and every…
D.
-------------------------------------------------------------------------------
OQ.03.2017.B.004
An 87-year-old old woman is diagnosed with depression and has begun Paroxetine (seroxate) therapy. Which
of the following is important to follow up on?
A. Sodium
B. Potassium
C. Chloride
D. Magnesium
E. TSH
-------------------------------------------------------------------------------
OQ.03.2017.B.005
A 92-year-oldold man has become increasingly slow and apathic, over the last month, he has lost interest in
eating and drinking, feels depressed and answers many questions with “I don’t know”. Up until the last
month he was functioning well, was in a good mood, mood, and even drove his car. There is no history of fall or
head trauma. Routine physical examination and laboratory tests are normal. Which of the following is the
most likely diagnosis?
A. Frontal dementia
B. Stroke
C. Alzheimer’s dementia
D. Pseudo dementia
E. A systematic tic illness, such as severe flu
-------------------------------------------------------------------------------
OQ.03.2017.B.006
Which of the following is not true regarding cognitive behavioral therapy?
A. Very effective in anxiety disorders
B. Characterized by vagueness of the patient-therapist
patient relationship
C. Time limited
D. Includes identification of thought distortions
E. Includes home work and exercises
-------------------------------------------------------------------------------
OQ.03.2017.B.007
drmada.com ‫ הגרסה הכי מעודכנת תמיד אפשר לתרגל חינם ב‬.‫גרסה זו ראשונית‬

A 38-year-oldold lawyer, going through a divorce is troubled by the process, nervous, having trouble to fall
asleep, and suffers from stress and headaches. She is feeling distressed and manages to work and function
at home. Which of the following is the most likely diagnosis?
A. Acute stress disorder
B. Posttraumatic stress disorder
C. Adjustment disorder
D. Somatization disorder
E. Major depressive disorder
-----------------------------------------------------------------
-------------------------------------------------------------------------------
OQ.03.2017.B.008
A 27-year-old woman describes transient episodes when she suddenly feels while speaking that it is not
her talking. She becomes troubled and worried and asks herself “who in fact is talking?”. She describes
herself in those moments as if she is listening to someone else talk. These episodes usually appear in
social circumstances that trigger anxiety. Which of the following is the most likely diagnosis?
A. Depersonalization disorder
B. Depressive disorder
C. Dissociative identity disorder
D. Posttraumatic stress disorder
E. Narcissistic personality
nality disorder
-------------------------------------------------------------------------------
OQ.03.2017.B.009
A 35-year-oldold man with no history of psychiatric illness,illness, and who had never used drugs, tries cannabis for
the first time. With his friends encouragement he smoked two cannabis cigarettes, one after the other,
about 3 minutes per cigarette. 30 minutes later he felt tachycardia, a dry mouth, increasing anxiety, and a
feeling his throat closing and he is goi going
ng to suffocate and die. Which of the following is the most likely
diagnosis?
A. Hyperthyroidism, goiter
B. Brief psychotic disorder
C. Cannabis-induced
induced anxiety disorder
D. Cannabis withdrawal
E. First episode schizophrenia
-------------------------------------------------------------------------------
OQ.03.2017.B.010
Which of the following is part of the criteria for autism spectrum disorder diagnosis?
A. Disorganized speech characterized by unclear and parallel talk
B. Stereotypical
typical and repetitive movement
C. Broad and diverse areas of interest
D. Difficulty in keeping attentive and tendency to mistake a…. uncarefulness
E. Recurring thoughts, experienced as intrusive and unwanted
-----------------------------------------------------------------------------------------------
OQ.03.2017.B.11
In which of the following the pulse pressure is normal?
A. Shock class I
B. Shock class II
drmada.com ‫ הגרסה הכי מעודכנת תמיד אפשר לתרגל חינם ב‬.‫גרסה זו ראשונית‬

C. Shock class III


D. Shock class IV
-----------------------------------------------------------------------------------------------
OQ.03.2017.B.12
What is active heating in trauma?
A. Wiping and dying
B. Cover with heated sheets and blankets
C. Using a heating lamp
D. Heating the trauma room
-----------------------------------------------------------------------------------------------
OQ.03.2017.B.13
An NPO patient requires IV fluids maintenance. His weight is 70 kg. What is the appropriate fluid order
(ml/h)?
A. 90
B. 100
C. 110
D. 120
E. 135
-----------------------------------------------------------------------------------------------
OQ.03.2017.B.14
Which of the following is not used for treatment of hyperkalemia?
A. Ventolin inhalation
B. Calcium gluconate infusion
C. Insulin and glucose infusion
D. Ringer solution infusion
E. Kayexalate enemas
-----------------------------------------------------------------------------------------------
OQ.03.2017.B.15
Whish of the following IS NOT a complication of central line insertion?
A. Peritonitis
B. Pneumothorax
C. Bacteremia
D. Bleeding
E. Air emboli
-------------------------------------------------------------------------------
OQ.03.2017.B.16
What is keloid of the surgical scar?
A. scar infection
B. dehiscence and opening of the scar
C. sinking of the scar
D. hypertrophy of the scar
-------------------------------------------------------------------------------
OQ.03.2017.B.17
Which
hich of the following is the result of negative pressure wound therapy (NPWT)? )?
drmada.com ‫ הגרסה הכי מעודכנת תמיד אפשר לתרגל חינם ב‬.‫גרסה זו ראשונית‬

A. prolongation of wound healing


B. increase patient survival
C. slowing of wound granulation
D. increase in number of wound debridements required
-------------------------------------------------------------------------------
OQ.03.2017.B.18
Which
hich of the following patients requires perioperative steroid increase?
A. patient receiving prednisone up to 4 mg/d over last year
B. patient receivingving prednisone 30 mg over last week
C. patient receivingving prednisone 10mg on alternating
alternati days
D. patient treated with steroid ointment
E. patient receivingving prednisone 30mg/d over last month
-------------------------------------------------------------------------------
OQ.03.2017.B.19
Which
hich of the following is a risk factor for surgical site infection (SSI)?
A. age 50
B. decrease use of drains
C. prolonged hospitalization prior to surgery
D. short surgeries
-------------------------------------------------------------------------------
OQ.03.2017.B.020
Which
hich of the following decreases the risk for surgical site infection (SSI)?
A. antibiotic treatment for at least 2 days before surgery
B. maintaining glucose level below 200 mg/dl for the days following surgery
C. shaving surgical site the evening before surgery
D. hypothermia during surgery
-------------------------------------------------------------------------------
OQ.03.2017.B.021
Which of the following is a risk factor for more than 5% risk of cardiac complications following surgery?
A. aortic surgery
B. carotid surgery
C. prostate surgery
D. knee replacement surgery
-------------------------------------------------------------------------------
OQ.03.2017.B.022
A trauma patient responds to pain only, mumbles incoherently, and performs extension on motor
response. What is the Glasgow coma scale (GCS) score of this patient?
A. 5
B. 6
C. 7
D. 8
-------------------------------------------------------------------
-------------------------------------------------------------------------------
OQ.03.2017.B.023
Which of the following is an indication for annual screening with MRI?
drmada.com ‫ הגרסה הכי מעודכנת תמיד אפשר לתרגל חינם ב‬.‫גרסה זו ראשונית‬

A. A woman following treatment for invasive ductal carcinoma (IDC)


B. A woman following breast augmentation
C. A life time risk for breast cancer of 10% according to genetic ic and risk factors assessment
D. BRCA 2 carrier
-------------------------------------------------------------------------------
OQ.03.2017.B.024
An 80-year- old male arrives to the ER complaining of nausea and vomiting in the last day. Abdominal
CT scan demonstrates air in the gallbladder, fluid levels, and small bowel distention with collapsed distal
bowel. What will the management ement of this patient include?
A. Enterotomy
B. Laparoscopic cholecystectomy
C. Gallbladder drainage
D. Enteroscopy
E. stricturoplasty
-------------------------------------------------------------------------------
OQ.03.2017.B.025
Which of the following breast lesions requires excision?
A. 1.5cm fibroadenoma
B. Atypical ductal hyperplasia(ADH)
C. Lobular carcinoma in situ (LCIS)
D. 2cm cyst
-------------------------------------------------------------------------------
OQ.03.2017.B.026
Which
hich of the following patients with gastrogastro-esophageal disease (GERD) has the highest success rate of an
antireflux procedure?
A. A patient with asthma secondary to GERD
B. A patient with dysphagia
C. A patient with weight loss
D. A patient with good response to proton pump inhibitors (PPI)
-------------------------------------------------------------------------------
OQ.03.2017.B.027
What is the most common histopathology in upper esophageal cancer?
A. Adenocarcinoma
B. Gastro-intestinal
intestinal stromal tumor
C. Squamous cell carcinoma
D. Neuroendocrine carcinoma
-------------------------------------------------------------------------------
OQ.03.2017.B.028
Which of the followingwing may be treated with omentopaxy (Graham’s patch)?
A. Duodenal perforation
B. Bleeding duodenal
C. Gastroesophageal reflux disease
D. Gastric carcinoma
-------------------------------------------------------------------------------
drmada.com ‫ הגרסה הכי מעודכנת תמיד אפשר לתרגל חינם ב‬.‫גרסה זו ראשונית‬

OQ.03.2017.B.029
What is the most common metabolic disorder following gastric resection?
A. Hypoglycemia
B. Iron deficiency anemia
C. Hypernatremia
D. Decrease in triglyceride
-------------------------------------------------------------------------------
OQ.03.2017.B.030
Why is hypothermia so harmful in trauma?
A. In trauma there is heat loss through the skin to the environment
B. Hypothermia contributes to coagulopathy
C. Hypothermia increases cellular ATP deficiency
D. Hypothermia causes capillary vasoconstriction
-------------------------------------------------------------------------------
OQ.03.2017.B.031
Which of the following is correct regarding cardiac tamponade following trauma?
A. Decreased venous return to the heart
B. Diagnosed by pericardiocentesis
C. Rarely affects patient’s survival
D. Cannot be caused by penetrating injury
-------------------------------------------------------------------------------
OQ.03.2017.B.032
An 8-year-oldold girl is brought by her parents to the ER following a fall from the window and injury to the
right thigh. Initial evaluation reveals a 5 cm wound in the medial aspect with active bleeding. What
should be included in the immediate management?
A. Direct wound pressure
B. Tourniquet placement above the wound
C. Wound dressing with external fixation fixat (Thomas)
D. Urgent surgery
-------------------------------------------------------------------------------
OQ.03.2017.B.033
A 30-year-oldold male is admitted to the ER 20 minutes following a knife stab to the right upper chest. On
admission he is fully alert and conscious,
conscious, pulse of 110 bpm, blood pressure is 80/60, and respiratory rate
is 20/minute. Chest X-ray ray demonstrates large hemothorax. Left chest drain is inserted and 1700 ml of
blood are immediately drained. What is the most appropriate next step?
A. Bronchscopy
B. Left thoracoscopy
C. Insertion of another chest drain
D. Left thoractomy
-------------------------------------------------------------------------------
OQ.03.2017.B.034
What is the most common etiology for lower gastrointestinal bleeding?
A. Colon cancer
B. Diverticulosis
drmada.com ‫ הגרסה הכי מעודכנת תמיד אפשר לתרגל חינם ב‬.‫גרסה זו ראשונית‬

C. Colitis
D. Angiodysplasia
E. Inflammatory bowel disease
-------------------------------------------------------------------------------
OQ.03.2017.B.035
A 42-year-oldold male has ulcerative colitis for 10 years years. Which of the following is not a definitive surgical
treatment?
A. total proctocolectomy with ileoctomy
B. restorative proctocolectomy with ileal pouch-anal
pouch anastomosis(IPAA)
C. total proctocolectomy with ileal reservoir(kock pouch)
D. subtotal abdominal colectomy
-------------------------------------------------------------------------------
OQ.03.2017.B.036
A 35 year old male is diagnosed with familial adenomatous
aden polyposis syndrome (FAP).
( Which of the
following is correct?
A. He needs to undergo a colonoscopy every 5 years.
B. his children have 25%chance of having FAP
C. the risk of colon cancer by the age of 50 reaches 100%
D. The surgical treatment of choice for this patient is total abdominal colectomy and ileorectal
anastomosis.
-------------------------------------------------------------------------------
OQ.03.2017.B.037
A 52 year old female is referred to the ER due to dark discharge from a laparotomy scar one month
following a laparotomy for small bowel obstruction. Abdominal CT demonstrates entero entero-cutaneic fistula.
Which of the following may help spontaneous closure of the fistula?
A. partial small bowell obstruction distal to the fistula
B. fistula secretion of more than 500ml/d
C. fistula tract longer than 20cm
D. epithelization of the fistula tract
E. previous radiation to the involved bowel segment
-------------------------------------------------------------------------------
OQ.03.2017.B.038
A 65-year-oldold male is admitted to the ER with right upper quadrant pain, fever of 39, 39 and chills. His
blood pressure is 100/40; he has jaundice of the sclera, sclera and RUQ tenderness. Labsabs- WBC of 15,000,
bilirubin 5, AST 200, GGT 1000. Abdominal bdominal ultrasound demonstrates cholelithiasis with no wall
thickening of the gallbladder. There here is intrahepatic bile dilation; CBD is 13 mm with a gallstone at its
end. What is the next step in management?
A. urgent surgery to remove the gallbladder and extract the choledochus stone
B. urgent endoscopic retrograde cholangiopancreatogram (ERCP)
C. NPO, IV fluids, IVantibiotics and elective surgery in the future
D. treatment with ursodeoxycholic acid to dissolve the stone and consider lithotripsy
-------------------------------------------------------------------------------
OQ.03.2017.B.039
drmada.com ‫ הגרסה הכי מעודכנת תמיד אפשר לתרגל חינם ב‬.‫גרסה זו ראשונית‬

A 58-year-oldold female undergoes appendectomy


appen for acute appendicitis. Final pathology identifies 1 cm
adenocarcinoma of the appendix, the base of the appendix is not involved. What hat is the appropriate
treatment?
A. right hemicolectomy
B. observation only
C. chemotherapy only
D. cecectomy
-------------------------------------------------
-------------------------------------------------------------------------------
OQ.03.2017.B.040
Inn which of the following cases a mammogram and ultrasound can be performed while omitting breast
MRI?
A. 50-year-old old woman with ILC
B. 40-year-old old woman with suspicious mass and dense breast on the mammography and ultrasound
C. 70-year-old old female with breast cancer metastatic to the axilla and fatty breast
D. routine follow up of 50-year year-old BRCA carrier
-------------------------------------------------------------------------------
OQ.03.2017.B.041
An 80-year-old male is diagnosed with gallstone ileus. Hee is in a moderate general condition due to
prolonged bowel obstruction, vomiting and dehydration. Which hich of the following will not be included in
the surgical treatment?
A. scan the bowel for another gallstone
B. cholecystectomy
cystectomy and closure of the duodenal fistula
C. enterotomy proximal to the stone, extraction of the stone, and closure of the enterotomy
D. milking of the stone via the ileocecal valve if possible
------------------------------------------------------------
-------------------------------------------------------------------------------
OQ.03.2017.B.042
A 55-year-oldold female is admitted to ER with fever, jaundice, and RUQ pain. On examination distinct
n examination:
RUQ tenderness with jaundice ndice of the skin and sclera. Labs-L elevated bilirubin, ALP,
ALP GGT. Which of the
following is the next step in the evaluation of this patient?
A. endoscopic retrograde cholangiopancreaticography (ERCP)
B. upper abdomen US
C. diagnostic laparoscopy
D. percutaneous cholecystostomy
-------------------------------------------------------------------------------
OQ.03.2017.B.043
A usually healthy 25-year-old old woman with no medications other than oral contraceptives undergoes
abdominal US for upper abdominal pain. There is an incidental 4 cm nodular solid lesion in the right lobe
of the liver. >>>>>>>>>protocol CT demonstrates
demonstrates the lesion as solid, fibrotic with a central scar, and 4
cm in size. What is the most probable diagnosis?
A. Metastasis from different tumor
B. Hepatocellular carcinoma
C. Atypical hemangioma
D. Focal nodular hyperplasia
E. Atypical biliary cyst
drmada.com ‫ הגרסה הכי מעודכנת תמיד אפשר לתרגל חינם ב‬.‫גרסה זו ראשונית‬

-------------------------------------------------------------------------------
OQ.03.2017.B.044
A 35-year-oldold man has a previous history of splenectomy following blunt trauma many years ago. The
patient presents to the ER with high fever, chills, muscle pain, pain, and general malaise. On the first day of
admission the patient develops severe sepsis with hypotension, coagulopathy and respiratory distress.
Which prior treatment could have prevented this clinical deterioration?
A. Annual flu vaccine
B. Prophylactic antibiotic treatment with regular low dose resprim
C. Low dose steroids treatment
D. Triple vaccination for streptococcus pneumonia,, haemophilus influenza, and Neisseria
meningitides
E. Intravenous immunoglobulin (IV Ig) treatment
-------------------------------------------------------------------------------
---------------------------------------------
OQ.03.2017.B.045 (same as OQ.10.2016.B.065 or similar)
What is the most common etiology for upper gastrointestinal bleeding?
A. Peptic ulcer disease
B. Gastritis
C. Mallory-Weiss tears
D. Dieulafoy’s lesion
E. Gastric malignancy
-------------------------------------------------------------------------------
OQ.03.2017.B.046
A 60-year-oldold hepatitis C carrier male with a history of variceal bleeding is admitted to the ER for
vomiting blood. His blood pressure on arrival is 110/70 and he has splenomegaly and ascites on physical
examination. His hematocrit is 25%. What is the first step in managing this patient before performing per
gastroscopy?
A. IV octreotide infusion
B. Treatment with beta blockers (i.e. propranolol)
C. PT test and cryglobuling infusion if increased
D. Empiric treatment with platelets
E. Gastro esophageal tamponade with Sengstaken-Blackmore tube
-------------------------------------------------------------------------------
OQ.03.2017.B.047
A usually healthy 45-year-old old male is admitted to the ER with epigastric pain. The patient describes
sudden onset pain on 10 am. On examination – heart rate of 110, blood pressure of 135/80, temperature is
38,5, and severe epigastric tenderness with positive rebound tenderness. Labs – WBC of 18,000 with 18%
bands, hemoglobin 14, lactate 3,5. Chest X-ray X demonstrates free air ir under the right diaphragm. What is
the next step in the management of this patient?
A. Gastrographin swallow test and decision accordingly
B. Urgent laparotomy
C. Urgent right thoracotomy
D. Urgent gastroscopy
-------------------------------------------------------------------------------
OQ.03.2017.B.048
drmada.com ‫ הגרסה הכי מעודכנת תמיד אפשר לתרגל חינם ב‬.‫גרסה זו ראשונית‬

All of the following may be complications of high output ileostomy, except:


A. Dehydration
B. Hyperalbuminemia
C. Renal failure
D. Electrolyte disturbance
E. Skin burn
-------------------------------------------------------------------------------
OQ.03.2017.B.049
A 55-year-oldold female is having abdominal pain, diarrhea, flushing, hypertension, and bronchospasm.
Chest and abdomen CT demonstrates small bowel mass with lung and liver lesions that are suspicious for
metastases. What is the most likely diagnosis?
A. Carcinoid
B. Lymphoma
C. Gastro intestinal tumor (GIST)
D. Adenocarcinoma
E. Leiomyosarcoma
-------------------------------------------------------------------------------
OQ.03.2017.B.050
A 25-year-oldold male with previous history of 2 perianal abscess episodes is complaining of anal
discomfort. On examination, right anal fistula with 2 external openings, left anal fistula with one opening,
skin tags, and 2 fissures of the anal canal are identified. What is the next step in the management of this
patient?
A. Colonoscopy
B. Evaluation under anesthesia and abscess drainage
C. Sphincterotomy for treating the anal fissure
D. Injection of the fistula with biologic glue to avoid injury to the sphincter
E. Diverting ileostomy for protection
-------------------------------------------------------------------------------
OQ.03.2017.B.051
A 42 year-old female arrives to clinic complaining of nipple discharge. On examination, no
lymphadenopathy, no evidence of breast masses bilaterally. Nipple appears pears normal but milk is extracted
when both nipples are pressed. What is the next step in the management of this patient?
A. Serum markers
B. Breast and axilla ultrasound
C. Prolactin level
D. As long as it’s bilateral discharge there is no need for further evaluation
E. Chest and head CT scan
-------------------------------------------------------------------------------
OQ.03.2017.B.052
A 25-year-oldold female with no family history of breast cancer is being evaluated for right breast mass. On
examination: palpable, mobile, smooth, regular mass, 2 cm, in the outer lower quadrant. Left breast is
normal and there is no lymphadenopathy bilaterally. What is the next step in the management of this
patient?
A. Follow up with repeat examination in 3 month
drmada.com ‫ הגרסה הכי מעודכנת תמיד אפשר לתרגל חינם ב‬.‫גרסה זו ראשונית‬

B. FNA
C. Mammography
D. Breast ultrasound
E. Breast MRI
-------------------------------------------------------------------------------
OQ.03.2017.B.053
A 64-year-oldold male has morbid obesity, COPD, diabetes, and heavy smoking. Pas surgical history is
positive for an exploratory laparotomy with surgical site infection during the postoperative course. He is
evaluated for an incisional hernia. All of the following are risk factors for developing incisional hernia
EXCEPT:
A. Obesity
B. Heavy smoking
C. COPD
D. Wound infection
E. Iron deficiency anemia
-----------------------------------------------------------------------------------------------
OQ.03.2017.B.54
A 45-year-oldold female is diagnosed with 2 cm nodule of the right thyroid lobe. In which of the following
scenarios a thyroid scan will be included in her evaluation?
A. Suppressed TSH
B. Ultrasound will identify another nodule in the left lobe
C. Thyroid scan should be performed in any case
D. Patient has a previous history of thyroiditis
E. Elevated thyroglobulin
------------------------------------------
-----------------------------------------------------------------------------------------------
OQ.03.2017.B.55 (OQ.10.2016.B.077)
A 45-year-oldold male has a history of Hodgkin’s lymphoma that was treated with radiation to the
mediastinum
ediastinum and neck. The patient now presents with a 3 cm mass in the left lobe of the thyroid. FNA
diagnoses malignancy. Which of the following is the most likely malignancy in this patient?
A. Follicular carcinoma
B. Hurtle cell carcinoma
C. Papillary carcinoma
D. Medullary carcinoma
E. Anaplastic carcinoma
-------------------------------------------------------------------------------
OQ.03.2017.B.056 (OQ.10.2016.B.079)
A 55-year-oldold female is diagnosed with primary hyperparathyroidism. What can be an indication for
surgery in this patient?
A. A calcium level of 11 mg/dL (NL 8.5-10.5) 8.5
B. Recent admission for nephrolithiasis
C. Osteopenia (bone density of the forearm of - 1.8)
D. Urinary calcium of 300 mg/24h (NL 100-300) 100
E. Family history of hypercalcemia
-------------------------------------------------------------------------------
drmada.com ‫ הגרסה הכי מעודכנת תמיד אפשר לתרגל חינם ב‬.‫גרסה זו ראשונית‬

OQ.03.2017.B.057
A 28-year-oldold male with Crohn’s disease has been treated with steroids until 4 month ago and is currently
treated with Rafassal. He is now admitted for resection of the terminal ileum and the cecum. The morning
following surgery the patient complains of abdominalabdominal pain, nausea and vomiting. His blood pressure is
80/50, he is treated with IVF and IV antibiotics (rocephin+flagyl). What should be added to this patient’s
treatment?
A. Hydrocortisone
B. 4th generation cephalosporin
C. IV insulin
D. Noradrenalin
E. Dexamethasone
-------------------------------------------------------------------------------
OQ.03.2017.B.058
Hepatocellular carcinoma is epidemically associated with aassociated ssociated with all of the following EXCEPT:
A. Viral infection with hepatitis A
B. Viral infection with hepatitis B
C. Viral infection with hepatitis C
D. Wilson’s disease
E. Alcoholic hepatitis
-------------------------------------------------------------------------------
OQ.03.2017. B.059 (02.01.PreTest
02.01.PreTest-Surgery-13e.07.361)
A 61-year-oldold female with a history of unstable angina is complaining of bloody vomiting following
hiccups and vomiting after a night of extensive drinking. Endoscopy demonstrates longitudinal tears in
the gastro-esophageal bleeding. Which of the following is the next most
esophageal junction that are not actively bleeding.
appropriate step in management?
A. Angiography and embolization
B. Balloon tamponade
C. Exploratory laparotomy, gastrostomy, and over over-suturing of the tear
D. Systemic vasopressin infusion
E. Conservative management
-------------------------------------------------------------------------------
OQ.03.2017.B.060 (02.01.PreTest-Surgery-13e.04.182)
(02.01.PreTest
A 58-year-oldold male is brought to the ER following a fall from a 3m ladder. On examination – vital signs
are normal, no respiratory distress, multiple right rib fractures. Chest X ray demonstrates right
X-ray
hemothorax and a thoracostomy tube are inserted. 700 mll of blood are drained immediately and over the
next 4 hours the drain continues to drain another 300 ml/h. Which of the following is the definitive
treatment in this patient?
A. Platelets
B. Fresh frozen plasma
C. Insertion of another chest drain
D. Thoracotomy
E. Observation
-------------------------------------------------------------------------------
OQ.03.2017.B.061 (06.06.Blueprints-QaA-for-STEP-2.03.01.001)
(06.06.Blueprints
drmada.com ‫ הגרסה הכי מעודכנת תמיד אפשר לתרגל חינם ב‬.‫גרסה זו ראשונית‬

An 8-year-oldold girl is brought in to the hospital while actively seizing. She has been hospitalized many
times before for status epilepticus and is receiving valproic acid at home to control seizures. The first step
in management of this patient is:
A. Administer ter 20 mL/kg 0,9% normal saline
B. Establish secure IV access and administer an anticonvulsant
C. Stabilize airway and provide 100% oxygen
D. Administer activated charcoal via nasogastric tube
-------------------------------------------------------------------------------
OQ.03.2017.B.062 (06.06.Blueprints-QaA-for-STEP-2.03.01.009)
(06.06.Blueprints
month old infant was delivered at 33 weeks gestation. The neonatal course was complicated by grade
A 6-month
II intraventricular hemorrhage on the left and initial slow feeding. The baby has been growing well but is
noted to have weakness on the right side and increased increased deep tendon reflexes in the right extremity. A
cerebral palsy is suspected. What is the diagnosis?
A. Diplegia
B. Hemiplegia
C. Extrapyramidal cerebral palsy
D. Quadriplegia
E. Dystonic cerebral palsy
-------------------------------------------------------------------------------
OQ.03.2017.B.063 (06.06.Blueprints-QaA-for-STEP-2.03.01.019)
(06.06.Blueprints
A 12-year-oldold female is admitted because of profuse, watery diarrhea that last 72 hours. Fluid
management has been difficult because the patient is noted to be hypertensive despite massive amounts of
diarrhea and marginal urine outputoutput.. On physical examination: she appears healthy but has multiple, brown
pigmented lesions on n the extremities, back and trunk. What of the following explains the findings?
A. Severe anxiety with neurodermatitis relate to hospitalization
B. Adrenal carcinoma with associated catecholamine release
C. Tachycardia and hypertension due to hyperthyroidism
D. Renovascular
scular hypertension secondary to neurofibromatosis
E. Tuberous sclerosis with intrarenal and intracardiac tubercles
-------------------------------------------------------------------------------
OQ.03.2017.B.064 (06.06.Blueprints-QaA-for-STEP-2.03.02.051)
(06.06.Blueprints
A 4-year-old male presents with 6 days of fever fever, erythematosus tongue and lips, nonexudative,
none bulbar
conjunctivitis, cervical lymphadenopathy and generalized maculopapular rash. A diagnosis of Kawasaki
disease is made. Which is the best therapy?
therapy
A. Alfa interferon (100 units IV) and Prednisone (2 mg/kg/day PO)
B. Prednisone (2 mg/kg/day) and Nafcillin (50 mg/kg/day IV)
C. Nafcillin (50 mg/kg/day IV) and Alfa interferon (100 units IV)
D. Nafcillin (50 mg/kg/day IV) and IVIG (2 gr/kg)
E. IVIG (2 gr/kg) and aspirin (100 mg/kg/day PO)
-------------------------------------------------------------------------------
OQ.03.2017.B.065 (03.07.Rudolphs-Pediatrics-22e.03.004)
(03.07.Rudolphs
A 16-year-oldold girl has been admitted with diagnosis of anorexia nervosa. At the last 6 month she has
placed herself on a calorie-restricted
restricted diet. One month prior to admission she had become irritable and had
a depressed mood. Over the last week she has also als developed diarrhea and an erythematosus rash that is
drmada.com ‫ הגרסה הכי מעודכנת תמיד אפשר לתרגל חינם ב‬.‫גרסה זו ראשונית‬

accentuated by sunlight exposure. The skin in the erythematosus area is rough, cracked and thick. Which
vitamin deficiency in this patient?
patient
A. A
B. B3
C. B12
D. D
E. E
-------------------------------------------------------------------------------
OQ.03.2017.B.066 (SIMILAR to 03.07.Rudolphs-Pediatrics-22e.03.016)
03.07.Rudolphs
A 10-day-old former 3.02-kg up. The infant is breast-
kg full term infant presents for a routine check-up.
feeding. He was discharged at 2 days of age from the nursery. His discharge weight was 2.85 kg. At 4
days of age his weight reached a nadir of 2.65 kg. Parents report that his stool transitioned from
meconium to yellow-mustard colored by 3 days of age. He is now 3.03-kg kg today at his 10-day-old
10 visit.
He has four wet diapers per day. Which of the following is an alarming sign for inadequacy of his breast- breast
feeding?
A. Regaining birth weight by 10 days of age
B. Loss of 12% of birth weight
C. Transition to mustard-coloredcolored stool by 3 days of age
D. Presence of four wet diapers per day
-------------------------------------------------------------------------------
OQ.03.2017.B.067 ( 03.07.Rudolphs-Pediatrics-22e.05.003)
03.07.Rudolphs
A full-term,
term, male neonate is delivered vaginally. One application of vacuum extractor to the neonate’s
head was required to facilitate delivery. On examination at 12 hours of age, the baby has a well well-
circumscribed, fluctuating mass over the parietal skull that does not cross suture lines. The overlying skin
is erythematous but intact. He is active and alert and rest of the examination is unremarkable. His
capillary hematocrit is 45%. Which step is next indicated in the management of this neonate?
A. Intravenouss ampicillin and cefotaxime
B. Serum bilirubin levels
C. Aspiration of the mass with a tuberculin syringe
D. Topical bacitracin ointment to the mass
E. Transfusion of packed red blood cells
-------------------------------------------------------------------------------
OQ.03.2017.B.068 (03.07.Rudolphs-Pediatrics-22e.05.006)
(03.07.Rudolphs
A full-term, 9-week-old follow up appointment. She was delivered vaginally
old female infant is seen for a follow-up
from the breech presentation. In the nursery, hip examination was unremarkable bilaterally in Ortolani
and Barlow maneuvers. Physical examination at this visit shows normal movement of the lower
dysplasia of the hip (DDH) in
extremities. What is the appropriate screening strategy for developmental dysplasia
this infant?
A. Repeat Ortolani and Barlow maneuvers
B. Comparison of the lower extremity length
C. Ultrasonography of both hips
D. Radiographs of both hips
E. CT scanning of both hips
-------------------------------------------------------------------------------
drmada.com ‫ הגרסה הכי מעודכנת תמיד אפשר לתרגל חינם ב‬.‫גרסה זו ראשונית‬

OQ.03.2017.B.069
A male infant is born at 26 weeks gestation (birth weight 800 gr) develops tachycardia and abdominal
distention prior to passing a blood
blood-tinged stool. His examinationination reveals decreased tone and activity,
tenderness on palpation of the abdomen, and paucity of abdominal sounds. His abdominal X X-ray shows
diffuse pneumatosis intestinalis without pneumoperitoneum or portal venous air. What is the most likely
diagnosis?
A. Meconium ileus
B. Necrotizing enterocolitis
C. Pyloric stenosis
D. Intestinal malrotation with volvulus
E. Gastroenteritis
-------------------------------------------------------------------------------
OQ.03.2017.B.070 (03.07.Rudolphs-Pediatrics-22e.06.003)
(03.07.Rudolphs
Which hormone is the most influential in affecting breast development during puberty?
A. Progesterone
B. Thyroxin
C. Growth hormone
D. Insulin
E. Estrogen
-------------------------------------------------------------------------------
OQ.03.2017.B.071 (03.07.Rudolphs-Pediatrics-22e.07.012)
(03.07.Rudolphs
The parents of a 10 years old are concerned that he is not doing well in school. He is a healthy, active 10 10-
old boy with no history of developmental delays. He states that he enjoys school and likes to read.
year-old
His teacher describes him as an active participant in class, engaged, and cooperative. He has difficulty
completing his homework assignments, particularly for English class, and his grades have decreased
recently. He has always struggled with penmanship. What is the most likely diagnosis?
A. Dyslexia
B. Dyscalculia
C. Dysnomia
D. Dysgraphia
E. Pragmatic language disorder
-------------------------------------------------------------------------------
OQ.03.2017.B.072
Which of the following is associated with frequent crying (colic) in an infant that is 3 months old?
A. Brest feeding
B. Gastroesophageal reflux
C. Lactose intolerance
D. Cow's mild allergy
E. Normal behavior of the infant
-------------------------------------------------------------------------------
OQ.03.2017.B.073 (03.07.Rudolphs-Pediatrics-22e.07.017)
(03.07.Rudolphs
A 15 months old female is seen in a well-child
well clinic. Mom is concerned that she only has 5 to 10 words
and is not putting words together. She points to body parts by name and follows 11-step commands
without gestures. What is the most appropriate next step?
drmada.com ‫ הגרסה הכי מעודכנת תמיד אפשר לתרגל חינם ב‬.‫גרסה זו ראשונית‬

A. Referral to audiology
B. Referral to speech-language
language pathologi
pathologist
C. Comprehensive developmental assessment
D. Reassurance
E. CT scan of head
-------------------------------------------------------------------------------
OQ.03.2017.B.074 (03.07.Rudolphs-Pediatrics-22e.09.037)
(03.07.Rudolphs
A 13 years old with refractory acute mylogenous leukemia (AML) and chronic pain is started on an
opioid treatment. Which medication has a slow elimination phase that may lead to drug toxicity?
A. Morphine
B. Hydromorphone
C. Fentanyl
D. Oxycodone
E. Methadone
-------------------------------------------------------------------------------
OQ.03.2017.B.075 (03.07.Rudolphs-Pediatrics-22e.11.027)
(03.07.Rudolphs
A 2-week-old old neonate is admitted with sepsis. Blood cultures reveal sepsis due to Escherichia coli ( E.
coli ). On physicalcal examination she is lethargic, skin appears icteric, and she has hepatomegaly. Testing
on urine reveals the presence of nonglucose reducing substances. What is the most likely underlying
diagnosis?
A. Glycogen storage disease
B. MSUD
C. Organic acidemia
D. Galactosemia
E. Urea cycle defect
-------------------------------------------------------------------------------
OQ.03.2017.B.076 (03.07.Rudolphs
03.07.Rudolphs-Pediatrics-22e.12.023)
Tay Sachs disease is an autosomal recessive degenerative neurologic disease. It is relatively common in
French speaking immigrants
the Cajun population of Louisiana. Cajuns descend from a group of French-speaking
(Acadians) from eastern Canada in the 18th century. It is postulated that one of them was a man who was
a carrier for the Tay Sachs mutation. His descendants carry the mutation and intermarriage has led to
appearance of the disease. This phenomenon of a mutation originating with a single individual is known
as:
A. Consanguinity
B. Endogamy
C. Decreased fitness
D. Founder effect
E. Genetic drift
-------------------------------------------------------------------------------
---------------------------------------------------------------------
OQ.03.2017.B.077 (03.07.Rudolphs-Pediatrics-22e.13.012)
(03.07.Rudolphs
Patients with disorders of B-cell cell development are susceptible to infections from encapsulated organisms
and enteroviral infections. These patients typically become symptomatic at what age?
A. In the second decade of life
B. Between the ages of 5 and 10
drmada.com ‫ הגרסה הכי מעודכנת תמיד אפשר לתרגל חינם ב‬.‫גרסה זו ראשונית‬

C. In the first three months of life


D. Between the first 6 and 12 months of life
E. In the immediate postpartum period
-------------------------------------------------------------------------------
OQ.03.2017.B.078 (03.07.Rudolphs-Pediatrics-22e.13.018)
(03.07.Rudolphs
Patients with IgA deficiency should be given which of the following with caution ?
A. Washed packed RBCs
B. IVIG
C. GM-CSF
D. Pneumococcal Immunization
E. Folic acid supplementation
-------------------------------------------------------------------------------
OQ.03.2017.B.079 (03.07.Rudolphs
03.07.Rudolphs-Pediatrics-22e.14.009)
A 10-year-oldold girl was stung by a hornet 20 minutes prior to arrival at the emergency department, and
now presents with wheezing, generalized urticaria, tongue angioedema, and repeated episodes of
vomiting. What is the most important initial medical therapy to administer in this setting?
A. IV diphenhydramine
B. IV fluids
C. IV methylprednisolone
D. IM epinephrine
E. Inhaled albuterol
-------------------------------------------------------------------------------
OQ.03.2017.B.080
All of the following reflexes are assessed in determination of brain death except
A. Papillary
B. Oculocephalic
C. Corneal
D. Babinsky
E. Gag
-------------------------------------------------------------------------------
OQ.03.2017.B.081
What is the most common location for insertion of the intraosseous needle during treatment of an acutely
ill pediatric patient in the emergency department?
A. Radius
B. Ulna
C. Tibia
D. Femur
E. Iliac crest
-------------------------------------------------------------------------------
OQ.03.2017.B.082 (03.07.Rudolphs-Pediatrics-22e.15.004)
(03.07.Rudolphs
A 3-year-oldold child is brought to the emergency room with fever to 39°C and refusal to bear weight for the
appearing and on exam has a swollen, tender right knee with surrounding
last several hours. She is ill-appearing
erythema. Radiographs of the right knee demonstrate
demonst an effusion. Which of the following tests would
most likely to ensure a definitive diagnosis?
drmada.com ‫ הגרסה הכי מעודכנת תמיד אפשר לתרגל חינם ב‬.‫גרסה זו ראשונית‬

A. Blood culture
B. Bone culture
C. MRI of the knee
D. Synovial fluid culture
E. Ultrasound of the knee
-------------------------------------------------------------------------------
OQ.03.2017.B.083
A young girl is hospitalized with papulo vesicular rash diagnosed as chicken pox. On the 4th day of the
papulo-vesicular
exanthema, her rash seems to be worsening and more painful. She now has a fever of 39.2 C . What is the
next best treatment?
A. Acyclovir
B. Antibiotics
C. IVIG
D. Topical hydro cortisone
-------------------------------------------------------------------------------
OQ.03.2017.B.084 (03.07.Rudolphs-Pediatrics-22e.17.034)
(03.07.Rudolphs
A 5-year-oldold boy presents with foul smelling diarrhea along with mild abdominal pain and loss of
foul-smelling
appetite for 2 weeks. Evaluation of the patient’s stool sample taken that day is positive for Giardia lamblia
by EIA. The best treatment for the patient’s condition would be:
A. TMP sulfa
B. Metronidazole
C. Ciprofloxin
D. Azithromycin
E. Paromomycin
-------------------------------------------------------------------------------
----------------------------------------------------------------------------
OQ.03.2017.B.085 (03.07.Rudolphs-Pediatrics-22e.17.036)
(03.07.Rudolphs
A 17 years old female is diagnosed at 16 weeks of pregnancy via ATB titers as likely having contracted
Toxoplasmosis gondii during gestation.
gestation What is the most likely way that she has acquired this infection?
A. Blood transfusion
B. Food containing cyst
C. Ingested house cat feces
D. Municipal water supply
E. Cat fleas containing larvae
-----------------------------------------------------------------------------------------------
OQ.03.2017.B.86 (03.07.Rudolphs-Pediatrics-22e.17.068)
(03.07.Rudolphs
A 1-year-old, unimmunized infant is admitted with difficulty breathing and “fits” of coughing associated
with whooping for the last two days. Which of the following is the most complication of this disease?
A. Death
B. Rib fracture
C. Pneumonia
D. Seizures
E. Encephalopathy
-----------------------------------------------------------------------------------------------
----------------------------------------------------------------------------------
OQ.03.2017.B.87
drmada.com ‫ הגרסה הכי מעודכנת תמיד אפשר לתרגל חינם ב‬.‫גרסה זו ראשונית‬

A 14-year-oldold boy was admitted to the pediatric ward yesterday for periorbital cellulitis of the right eye.
On history he frequently has sinus problems and seasonal allergies. The patient was started on oral
clindamycin yesterday morning, but today shows signs of mild proptosis and chemosis that were not
present at admission. The change is symptoms can be explained due to the appearance of which of the
following?
A. Orbital cellulitis
B. Sinus vein thrombosis
C. Brain abscess
D. Bacterial meningitis
E. Sphenoidal sinusitis
-----------------------------------------------------------------------------------------------
OQ.03.2017.B.088
A 16-year-oldold Bedouin male presents with fever, hepatosplenomegaly, lymphadenopathy and arthritis.
The mother reports that they make their own goat cheese. A zoonotic infectious disease is suspected.
Which of the following is the best treatment option?
A. Ceftriaxone
B. Doxycycline
C. Cephalexin
D. Azithromycin
E. Erythromycin
-----------------------------------------------------------------------------------------------
OQ.03.2017.B.089 (03.07.Rudolphs-Pediatrics-22e.18.016)
(03.07.Rudolphs
A 16-year-oldold boy presents with complaint of scalp scaling and mild itching for the last 6 months. He
shampoos every other day. On examination he has scaling and erythema in the alar grooves and medial
brows. He has no alopecia. What is the most likely diagnosis?
A. Tinea capitis
B. Seborrheic dematitis
C. Psoriasis
D. Atopic dermatitis
E. Contact dermatitis
-----------------------------------------------------------------------------------------------
OQ.03.2017.B.090 (03.07.Rudolphs-Pediatrics-22e.19.006)
(03.07.Rudolphs
What is the most important physical examination finding diagnosing otitis media with effusion?
A. Dull tympanic membrane appearance
B. Absent light reflex
C. Decreased tympanic membrane mobility
D. Obscure ossicles
E. Air bubbles behind the tympanic membrane
-----------------------------------------------------------------------------------------------
OQ.03.2017.B.091 (03.07.Rudolphs-Pediatrics-22e.19.018)
(03.07.Rudolphs
A 6-year-oldold boy presents with intermittent high fevers, pharyngitis, lymphadenopathy and mouth
aphthae. His throat has been cultured several times and has never grown group A streptococcus. What is
the most likely diagnosis?
A. Familial mediterranean fever ((FMF)
B. PFAPA
C. Lymphoma
D. Kawasaki disease
E. Viral pharyngitis
-----------------------------------------------------------------------------------------------
drmada.com ‫ הגרסה הכי מעודכנת תמיד אפשר לתרגל חינם ב‬.‫גרסה זו ראשונית‬

OQ.03.2017.B.092 (03.07.Rudolphs-Pediatrics-22e.21.008)
(03.07.Rudolphs
A 15-years-oldold female complains of daily penumbilical abdominal pain without identifiable trigger for the
past 12 months. Review of growth records indicates that she has been between the 50th and 75th
percentiles for body mass index over the past several years. years. Physical examination is unremarkable. What
additional finding suggests an organic cause for her pain rather than a functional etiology?
A. Pain is severe enough to cause significant school absenteeism
B. History of irritable bowel syndrome is first-degree
first relatives
C. Sensation of incomplete evacuation after passage of stools
D. Complaint of fatigue and headache in addition to abdominal pain
E. Awakening from sleep to pass loose stool at night
-------------------------------------------------------------------------
-----------------------------------------------------------------------------------------------
OQ.03.2017.B.093 (03.07.Rudolphs-Pediatrics-22e.21.010)
(03.07.Rudolphs
A 20-month-old old female is seen with a 4-month
4 history of 3-5 5 watery stools per day. She is in the 60th
percentile for weight and 75th percentile for height. Her mother describes her as a playful child and has
no concern other than the diarrhea. Dietary history reveals that the girl eats 3 meals and at least 2 small
snacks and drinks 2-33 cups of apple juice in addition to 350 cc of milk each day. Which is the most
appropriate step in management of this patient?
A. Determination of stool electrolytes and osmolality
B. Determination of stool pH and reducing substances
C. Stool culture
D. Two-week
week trial of glucose-free
glucose diet
E. Two-week trial of juice--free diet
-----------------------------------------------------------------------------------------------
OQ.03.2017.B.094 (03.07.Rudolphs-Pediatrics-22e.21.011)
(03.07.Rudolphs
A 32-month-old old girl is seen because of constipation. Which finding in the child’s medical history
suggests an organic cause of constipation rather than functional cause?
A. Daily fecal soiling in undergarments
B. Two urinary tract infections within the last 6 months
C. Involuntary encopresis
D. Small-caliber,
caliber, thin appearing stools
E. Episodes of posturing with crossing of the legs and screaming
-----------------------------------------------------------------------------------------------
OQ.03.2017.B.095 (03.07.Rudolphs-Pediatrics-22e.22.007)
(03.07.Rudolphs
A previously healthy 9-year-old old male presents with jaundice and is found to have a total bilirubin of 11
mg/dL with a direct component of 8.5 mg/dL.Which test is the most useful indicator of hepatic synthetic
function?
A. Cortisol
B. Total and free carnitine and acyl carnitine
C. Complete blood count with platelets
D. Prothrombin time
E. Antinuclear antibody
-----------------------------------------------------------------------------------------------
OQ.03.2017.B.096
A one year old boy was found to have iron deficiency anemia on a routine screening . In which
circumstance a parenteral iron on administration is considered?
considered
A. Abdominal pain associated with oral intake .
B. Malabsorption .
C. Concomitant folate deficiency .
drmada.com ‫ הגרסה הכי מעודכנת תמיד אפשר לתרגל חינם ב‬.‫גרסה זו ראשונית‬

D. Vomiting due to gastroesophageal reflux .


-----------------------------------------------------------------------------------------------
OQ.03.2017.B.097
A 5 year old boy presents with pallor and fatigue . Two weeks ago he was diagnosed with upper
extremely pale child . Temperature 36.7C , heart rate 160 per
respiratory infection . On examination : an extremely
minute , respiratory rate 26 per minute and blood presure 110/54 . CBC : 1300 mm3 , hemoglobin 5.4g/dl 5.4g/dl,
hematocrit 16.2% , platelets 134,000 . Direct antiglobulin test positive for IgG . What is the best treatment
for this patient ?
A. Methotrexate
B. Corticosteroids
C. IVIG
D. Splenectomy
E. Antithymocyte antiglobulin
-----------------------------------------------------------------------------------------------
OQ.03.2017.B.098 (03.07.Rudolphs-Pediatrics-22e.24.018)
(03.07.Rudolphs
A 16 year old girl in the middle of therapy for acute lymphoblastic leukemia present with new fever and
cough . On examination the patient is noted to be tachypenic . Chest X-ray X ray reveals bilateral interstiail
pneumoniaa . Upon farther questioning she admits that she has not taken her trimethoprim trimethoprim-
sulfamethoxazole in months . Which organism is most likely causing the pneumonia ?
A. Hemophilus influenza .
B. Aspergillus .
C. Mycoplasma pneumonia
D. Pneumocystis carinii
E. Streptococcus pneumonia
-----------------------------------------------------------------------------------------------
OQ.03.2017.B.099
An 11 year old patient with newly diagnosed chronic kidney disease , is admitted due to asthma
exacerbation . Her height is significantly below the 3rd percentile for her age . What is the most
appropriate initial step in the management for her growth retardation ?
A. Immediate start of growth hormone therapy .
B. Correction of metabolic acidosis if present .
C. Endocrinology ogy consultation
D. Measurement of serum growth
-----------------------------------------------------------------------------------------------
OQ.03.2017.B.100 (03.07.Rudolphs-Pediatrics-22e.25.010)
(03.07.Rudolphs
A 13 year old , previously healthy , child present with 1 day history of gross hematuria , dysuria and
urinary urgency . A plain abdomianl film demonstrates nephrolithiasis . He is started on IV hydration and
soon after , he passes a stone and his symptoms resolve . What is the most appropriate next step ?
A. Non enhanced CT scan of the abdomen and pelvis
B. Urological consultation for shock wave lithotripsy
C. Oral thiazide diuretics
D. Urine collection for calcium , oxalate , urate and citrate
E. Calcium - restricted diet
-------------------------------------------------------------------------------
drmada.com ‫ הגרסה הכי מעודכנת תמיד אפשר לתרגל חינם ב‬.‫גרסה זו ראשונית‬

OQ.03.2017.B.101
An 8 year old with a history of sickle cell nephropathy is admitted with sickle cell pain crisis .
Electrolytes reveal potassium of 4.3meq\l
4.3meq and on EKG demonstrates peaked T-waves waves . What is the most
appropriate next step ?
A. Intravenous calcium gluconte
B. Oral kayexelate
C. Echocardiography
D. Intravenous sodium bicarbonte
E. Intravenous frrosemide
-----------------------------------------------------------------
-------------------------------------------------------------------------------
OQ.03.2017.B.102 (03.07.Rudolphs-Pediatrics-22e.26.013)
(03.07.Rudolphs
Which of the following forms of congenital heart lesion is most common?
A. Truncus arteriosus
B. Total anomalous pulmonary venous connection
C. Ventricular septal defect
D. Hypoplastic left heart syndrome
E. Aortic stenosis
-------------------------------------------------------------------------------
OQ.03.2017.B.103 (03.07.Rudolphs-Pediatrics-22e.26.021)
(03.07.Rudolphs
A 7 weeks old previously healthy, infant is seen because of a 2 weeks history y of irritability while feeding
feeding,
poor oral intake and tachypnea. She is afe ile. On exam respiratory rate is 60 per minute, without
afebrile.
retraction,, heart rate is 170 per minute,
minute, liver edge is 4cm below the costal margin. She has an active
precordium and d a new pan systolic murmur at the apex. The ECG show sinus tachycardia, deep Q wave in
leads I, aVL, and V4-6. 6. What is the most likely diagnosis?
A. Glycogen storage disease
B. Pericarditis
C. Supraventricular tachycardia
D. Anomalous origin of left coronary artery
E. Endocardial fibroelastosis
-------------------------------------------------------------------------------
OQ.03.2017.B.104 (03.07.Rudolphs-Pediatrics-22e.26.024)
(03.07.Rudolphs
A 15 year old male is evaluated in the clinic . His father had recently drowed while swimming in a pool .
A couple of months ago , his father had had a fainting episode after running . The boy admits that he has
palpitations during sport activity , but he never fainted or had any
had some episodes of dizziness and palpitations
chest pain . Chest X-ray ray is normal . What changes on his ECG are most worris one ?
A. First degree AV block ( prolonged RR interval )
B. Premature atrial contraction
C. Prolonged QTc interval
D. Sinus arrhythmia
-------------------------------------------------------------------------------
OQ.03.2017.B.105 (03.07.Rudolphs-Pediatrics-22e.27.011)
(03.07.Rudolphs
A 2 year old boy comes to the ED with sudden onset of cough and respiratory distress . His father was
watching television and eating popcorn when he noted his son coughing on the floor . On exam :
wheezing is auscultated on the left lung . What is the most appropriate study to order ?
drmada.com ‫ הגרסה הכי מעודכנת תמיד אפשר לתרגל חינם ב‬.‫גרסה זו ראשונית‬

A. CT scan of the chest .


B. Arterial blood gasses
C. Complete blood count
D. Chest X-ray
E. Spirometry
-------------------------------------------------------------------------------
OQ.03.2017.B.106 (03.07.Rudolphs-Pediatrics-22e.27.017)
(03.07.Rudolphs
Which of the following physical findings most likely be present in a 2 year old child with primary ciliary
dyskinesia that has not yet been diagnosed ?
A. Promient digital clubbing
B. Bronchniectasis
C. Situs in versus
D. Rhinosinusitis since birth
E. Nasal polyps
-------------------------------------------------------------------------------
OQ.03.2017.B.107 (03.07.Rudolphs-Pediatrics-22e.27.013)
(03.07.Rudolphs
A 3 month old boy is seen because of stridor and respiratory distress . His parents report that he
developed a runny nose and harsh barky cough since yesterday . Which test would be most appropriate to
order at this time ?
A. CT scan
B. Neck X-ray
C. Chest X-ray
D. C-Reactive protein
E. Complete blood count
-------------------------------------------------------------------------------
OQ.03.2017.B.108 (03.07.Rudolphs-Pediatrics-22e.28.029)
(03.07.Rudolphs
Which of the following is the most comman cause of ambiguous genitalia in a female newborn ?
A. Turner syndrome
B. Congenital adrenal hyperplasia
C. Noonan syndrome
D. 5-alph
alph reductase deficiency
E. Klinefelter syndrome
-----------------------------------------
-------------------------------------------------------------------------------
OQ.03.2017.B.109 (03.07.Rudolphs-Pediatrics-22e.28.037)
(03.07.Rudolphs
A 4 year old girl is admitted to the hospital with diabetic ketoacidosis . Her weight is 20 kg . She received
2l of IV fluids initially when she arrived in the ER 6 hours ago . She had been feeling better , but now she
is more somnolent and difficult to ar ouse . Her blood glucose is 214 mg/dl , and she is on an insulin drip
arouse
and IV fluids at 2 times maintenance . Which is the most appropriate next step in the managment of this
patient ?
A. Order head CT scan
B. Administer bicarbonate
C. Intubate and hyperventilate late
D. Administer mannitol
E. Increase the IV fluid rate
drmada.com ‫ הגרסה הכי מעודכנת תמיד אפשר לתרגל חינם ב‬.‫גרסה זו ראשונית‬

-------------------------------------------------------------------------------
OQ.03.2017.B.110 (03.07.Rudolphs-Pediatrics-22e.29.014)
(03.07.Rudolphs
An 8 year old boy comes in with 2 days of left sided facial weakness . He awoke with it in the morning
and it has been stable since . On examination , the left side of his face moves significantly less than the
right . The forehead is involved in palsy . T he remainder of his neurologic examination is normall . What
The
is the localization of his facial weakness ?
A. Cortex
B. Muscle
C. Seventh nerve
D. Eighth nerve
E. Midbrain

S-ar putea să vă placă și